T1 Test

You might also like

Download as pdf or txt
Download as pdf or txt
You are on page 1of 77

.

IASBABA’S AIPTS 2024 - PRELIMS TEST 1 (ANCIENT HISTORY)

Q.1) With reference to ‘Palaeolithic Age’, the correct explanation for


consider the following statements Statement-I.
Statement I: In India this culture was co- c) Statement-I is correct and but
terminus with the Ice age or the Pleistocene Statement-II is incorrect.
period d) Statement-I is incorrect but
Statement II: Paleolithic sites are extensively Statement-II is correct.
developed in the alluvial plains of the Ganga
and Indus. Q.3) Consider the following pair is correctly
matched?
Which one of the following is correct in
respect of the above statements? 1. Early Harappan Phase:
Regionalization Era
a) Both Statement-I and Statement-II
2. Mature Harappan Phase: Integration
are correct and Statement-II is the
correct explanation for Statement-I. Era
b) Both Statement-I and Statement-II 3. Late Harappan Phase: Localisation
are correct and Statement-II is not Era
the correct explanation for
Statement-I. How many of the following pairs is/are
c) Statement-I is correct and but correctly matched?
Statement-II is incorrect. a) Only one pair
d) Statement-I is incorrect but b) Only two pairs
Statement-II is correct. c) Only three pairs
d) None of the pairs are correctly
Q.2) With respect to the ‘Stone Age in India’,
matched
consider the following statements

Statement I: The domestication of animals Q.4) With reference to the Mesolithic Age,
was observed for the first time in the consider the following statements:
Neolithic age.
Statement II: Burzahom is a Neolithic site 1. New forms of chipped stone tools
found in present day Kashmir valley. called ‘microliths’ were used.
2. The lives of Mesolithic people were
Which one of the following is correct in based on hunting, fishing and food
respect of the above statements? gathering.
3. Mahadaha in Uttar Pradesh is a
a) Both Statement-I and Statement-II
Mesolithic site.
are correct and Statement-II is the
correct explanation for Statement-I. How many of the statements given above
b) Both Statement-I and Statement-II is/are correct?
are correct and Statement-II is not
a) Only one
.
IASBABA’S AIPTS 2024 - PRELIMS TEST 1 (ANCIENT HISTORY)

b) Only Two d) 1, 2 and 4 only


c) All Three
d) None Q.7) Which of the following are correct
about Mohenjodaro?
Q.5) Consider the following features of 1. It literally means mound of dead.
Neolithic culture: 2. It is situated in the Larkana district of
Sindh, on river Indus.
1. Grinding and polishing of stone tools
3. The Great bath, assembly hall and
having sharper edges.
the great granary are found here.
2. Pottery and weaving.
4. Women cloths, bronze dancing girl,
3. Use of Iron tools for agriculture.
steatite statue of a bearded man,
Which of the above feature is/are correct? were some of the other findings.

a) 1 and 2 only Choose the correct answer using the code


b) 1 and 3 only below:
c) 2 and 3 only
a) 1, 2, and 4 only
d) 1, 2 and 3
b) 2, 3 and 4 only
c) 1, 3 and 4 only
Q.6) Consider the following statements
d) 1, 2, 3 and 4
about the town planning during Harappan
Civilization:
Q.8) ‘This archaeological site is in Rajasthan
1. The towns were divided into two and has discovered the evidence of the
parts, citadel for ruling class and world's oldest plowed agricultural field. It
lower town for common people. has also a location that has provided
2. Every house featured a bathroom, evidence of the first recorded earthquake. A
sometimes latrines also were found wooden furrow and seven fire altars in a row
on both the ground and first floors. have been discovered, implying the practice
3. No Citadel has been found in of sacrifice.’
Kalibangan.
Which of the following sites does it refers to?
4. Dholavira town in the Indus Valley
Civilization was divided into three a) Karanpura
divisions. b) Rangpur
c) Kalibangan
Which of the following sentences is/are
d) Dholavira
correct?

a) 1 and 2 only Q.9) Which of the following is the best


b) 2 and 3 only description of Citadel in context of Harappan
c) 1, 2 and 3 only Civilisation?
.
IASBABA’S AIPTS 2024 - PRELIMS TEST 1 (ANCIENT HISTORY)

a) The countryside area in the Q.12) Consider the following statement


Harappan Civilisation. about the seals found at the Harappan sites:
b) The area inhabited by the poor
1. Gold and ivory were used for making
people in the Harappan Civilisation.
these seals.
c) The walled and higher area of the
2. They were also used as a necklace.
Harappan Civilisation.
3. Even Rhinoceros and Crocodile were
d) The lower town of the Harappan
depicted on these seals.
Civilisation.
4. The purpose of producing seals was
solely artistic.
Q.10) Which of the following statement is
correct about the `Cire-perdue’ technique Choose the correct answer using the codes
practiced by the Harappans: given below:
a) It is used to make terracotta a) 1, 2 and 3 only
figurines. b) 1, 3 and 4 only
b) It is a method of metal casting. c) 1 and 4 only
c) It is a method of measurement. d) 1 and 3 and only
d) It is used in weaving cloths.
Q.13) Which of the following Harappan site
Q.11) Consider the following statements is located near to the Iranian border?
with reference to Indus Valley Civilization:
1. The people in the civilization a) Sutkagendor
frequently suffered from floods and b) Surkotada
thus had taken initiative for c) Kotla Nihangkhan
management of this disaster. d) Aalamgirpur
2. Dogs were not regarded as pets.
3. They did not build dams but they Q.14) Match the following with List-I
built canals to divert water for (Harappan sites) to List-II (Rivers):
irrigation.
List-I List-II
How many of the statements given above
is/are correct? (Harappan sites) (Rivers)
1. Lothal A. Indus
a) Only one 2. Kalibangan B. Sutlej
b) Only Two 3. Chanhu- C. Sabarmati
c) All Three Daro
d) None 4. Surkotada D. Ghaggar
5. Ropar E. Bhogava
.
IASBABA’S AIPTS 2024 - PRELIMS TEST 1 (ANCIENT HISTORY)

Choose the correct answer using the code Q.17) With reference to Evolution of Pottery
below in ancient India Match list I with list II.

a) 1-B, 2-E, 3-A, 4-D, 5-C


List I List II
b) 1-A, 2-D, 3-B, 4-C, 5-E Neolithic Northern
c) 1-D, 2-E, 3-B, 4-C, 5-A Age Black
d) 1-E, 2-D, 3-A, 4-C, 5-B Polished
Ware
Q.15) Which of the following statements are Indus Ochre-
correct with regard to the religious beliefs of Valley colored
Civilization pottery
Harrapan people?
Vedic Age Hand-made
1. The Harappan religion is normally pottery
termed as animism. Mauryan Painted Grey-
Period ware
2. The evidence of fire worship has
been found at Kalibangan and Lothal.
Select the correct answer using the codes
3. Harappan people had a belief in life
given below:
after death.
a) 1-A, 2-B, 3-C, 4-D
How many of the statements given above
b) 1-A, 2-C, 3-D, 4-B
is/are correct?
c) 1-C, 2-B, 3-D, 4-A
a) Only one d) 1-C, 2-D, 3-B, 4-A
b) Only Two
c) All Three Q.18) Which of the following statements
d) None is/are correct about Aryans?

1. The Battle of Ten Kings established


Q.16) Which of the following statement
the supremacy of Bharatas.
regarding Harappan civilization is correct?
2. The Purus, Kurus and Tritsu were
a) The Harrapan civilisation was rural tribal ruling clans among Aryans.
based. 3. The archaeological evidences show
b) The inhabitants of Harappan grew that they possessed horses, chariots
and used cotton. and some better arms made of
c) The inhabitants of Harappa had Bronze.
neither knowledge of copper nor
Choose the correct answer using the code
bronze.
below:
d) The standard Harappan seals were
made of terracotta. a) 1 and 2 only
b) 1 and 3 only
.
IASBABA’S AIPTS 2024 - PRELIMS TEST 1 (ANCIENT HISTORY)

c) 2 and 3 only Which one of the following is correct in


d) 1, 2 and 3 respect of the above statements?

a) Both Statement-I and Statement-II


Q.19) Consider the following pairs regarding
are correct and Statement-II is the
the characteristics of Matsya Puranas:
correct explanation for Statement-I.
1. Sarga: It describes the creation of the b) Both Statement-I and Statement-II
Universe. are correct and Statement-II is not
2. Pratisarga: It describes the various the correct explanation for
eras. Statement-I.
3. Vamshanucharita: The royal lineage c) Statement-I is correct and but
of Suryavanshis and Chandravanshis. Statement-II is incorrect.
d) Statement-I is incorrect but
How many of the following pairs is/are
Statement-II is correct.
correctly matched?

a) Only one pair Q.22) Consider the following statements


b) Only two pairs
Statement I: ‘Kassite’ and ‘Mitanni’
c) Only three pairs
inscriptions are located in present day
d) None of the pairs are correctly
Turkey.
matched
Statement II: Hittites, Mitannians and the
ruling class of the Kassites belonged to a very
Q.20) In the Rig Vedic period, who was
large ethno-linguistic group called 'Indo-
known as ‘Vshayapati’?
European'.
a) One who had great knowledge of
Which one of the following is correct in
Vedas.
respect of the above statements?
b) Head of a group of villages.
c) Local ruler who patronized Vedic a) Both Statement-I and Statement-II
worship and sacrifices. are correct and Statement-II is the
d) The one who headed the military correct explanation for Statement-I.
units. b) Both Statement-I and Statement-II
are correct and Statement-II is not
Q.21) Consider the following statements: the correct explanation for
Statement-I.
Statement I: Sruti is solely of divine origin
c) Statement-I is correct and but
and contains no specific concepts of law.
Statement-II is incorrect.
Statement II: In Smriti, the desire is more
d) Statement-I is incorrect but
towards understanding and interpreting the
Statement-II is correct.
oral tradition.
.
IASBABA’S AIPTS 2024 - PRELIMS TEST 1 (ANCIENT HISTORY)

Q.23) The following sentences describe the


text of the Vedic culture, ‘It contain some of Q.25) With respect to ‘Maharshi Badrayan’,
the earliest discussions about key consider the following statements
philosophical terms like atman (the self),
Statement I: He was the founder of the
Brahman (ultimate reality), and focus a lot
Vedanta system of philosophy.
on the philosophy about life, universe, self,
Statement II: He is considered as author of
body, sacrifice, etc. They emphasize that the
the Brahma sutras.
knowledge of the self or atman should be
acquired and that the relation of the atman Which one of the following is correct in
with the Brahman should be properly respect of the above statements?
understood.’
a) Both Statement-I and Statement-II
Which of the following Vedic literature are correct and Statement-II is the
describes the above paragraph? correct explanation for Statement-I.
b) Both Statement-I and Statement-II
a) Brahmanas
are correct and Statement-II is not
b) Upanishads
the correct explanation for
c) Vedanta
Statement-I.
d) Vedanga
c) Statement-I is correct and but
Statement-II is incorrect.
Q.24) Consider the following statement
d) Statement-I is incorrect but
about the status of women during Rig Vedic
Statement-II is correct.
Period:

1. The education of women was not Q.26) Consider the following statements
neglected. regarding Political Organisation during later
2. Parada system was prevalent in the Vedic period:
society.
3. Child marriage was not prevalent in Statement I: The Vidatha completely
the society. disappeared.
4. The women were not independent Statement II: The Samiti became more
persons in the eye of the law. important than the Sabha.
Which one of the following is correct in
Which of the following statement is/are respect of the above statements?
correct?
a) Both Statement-I and Statement-II
a) 1, 2 and 3 only are correct and Statement-II is the
b) 1, 2 and 4 only correct explanation for Statement-I.
c) 1, 3 and 4 only b) Both Statement-I and Statement-II
d) 1, 2, 3 and 4 are correct and Statement-II is not
.
.
.
.
IASBABA’S AIPTS 2024 - PRELIMS TEST 1 (ANCIENT HISTORY)

Select the correct answer using the code a) 1 and 2 only


given below: b) 1 and 3 only
c) 2 and 3 only
a) 1 and 2 only
d) 1, 2 and 3
b) 2 and 3 only
c) 1 and 3 only
Q.42) With reference to the causes of rise of
d) 1, 2 and 3
Buddhism and Jainism, Consider the
following statements:
Q.40) Consider the following statements
about Jainism after Mahavira: 1. Vedic religion had become highly
ritualistic.
1. The Theras were the chief preceptors
2. Jainism and Buddhism were taught in
of the Jain order after Mahavira.
Prakrit and Pali respectively, thus
2. The Purvas are the doctrinal texts of
were more accessible to the common
Jainism, which contained the original
man as compared to Sanskrit.
doctrines of Mahavira.
3. It was accessible to people of all
3. Samaiyapantha and Bisapantha are
castes.
jaina sects.
4. The Varna system had rigidified and
How many of the statements given above people of the lower castes led
is/are correct? miserable lives and Jainism and
Buddhism offered them an
a) Only one honorable place.
b) Only Two
c) All Three Which of the above statements is/are
d) None correct?

a) 1, 2 and 3 only
Q.41) Which of the following reasons
b) 1, 2 and 4 only
attributed to the decline of Buddhism in
c) 1, 3 and 4 only
India?
d) All of the above
1. Shift to the language of intellectuals
from the language of people. Q.43) Consider the following pairs is/are
2. Increased corrupt practices in rich correctly matched?
monasteries.
3. Atrocities on Buddhists by
School of Ideology
Brahmanical Kings. Thought
Samkhya Nature important than
Select the correct answer using the code
God
given below:
.
IASBABA’S AIPTS 2024 - PRELIMS TEST 1 (ANCIENT HISTORY)

Mimamsa Salvation through b) Indica


meditation c) Arthasastra
Vaisheshika Propounded the atom d) Deepavamsa
theory

How many of the following pairs is/are Q.46) Consider the following terms related
correctly matched? to Mahajanapadas:

a) Only one pair 1. Karmakaras - Labourers


b) Only two pairs 2. Kassakas - Small Landowners
c) Only three pairs 3. Grihapatis - Rich Landowners
d) None of the pairs are correctly 4. Shudras - Cultivators and artisans
matched
How many of the following pairs is/are
correctly matched?
Q.44) With reference to the influence of
foreigners on ancient Indian history, a) Only one pair
consider the following statements: b) Only two pairs
c) Only three pairs
1. Indo-Greek rulers were the first to
d) All four pairs are correctly matched
issue gold coins in India.
2. During the period of Kushan rulers,
Q.47) With reference to Mahajanapadas,
third Buddhist council was held in
consider the following statements:
India.
3. The calendar system started by Statement I: The Buddhist text, Anguttara
Shaka rulers is used by the Nikaya, gives the list of 16 Mahajanapadas.
government of India as an official Statement II: The Mahajanapadas extended
calendar. from the North-Western Pakistan to the
plains of Punjab.
How many of the statements given above
is/are correct? Which one of the following is correct in
respect of the above statements?
a) Only one
b) Only Two a) Both Statement-I and Statement-II
c) All Three are correct and Statement-II is the
d) None correct explanation for Statement-I.
b) Both Statement-I and Statement-II
Q.45) Which of the following text narrates are correct and Statement-II is not
Chandragupta’s accession to the throne of the correct explanation for
Magadha? Statement-I.

a) Mudrarakshasa
.
IASBABA’S AIPTS 2024 - PRELIMS TEST 1 (ANCIENT HISTORY)

c) Statement-I is correct and but Q.50) Consider the following statements


Statement-II is incorrect. regarding the ‘Mauyran Period’.
d) Statement-I is incorrect but
Statement I: The samaharta was the highest
Statement-II is correct.
officer in charge of assessment and the
sannidhata was the chief custodian of the
Q.48) Identify the center of learning with the state treasury.
help of given information: Statement II: The punch-marked coins were
completely disbanded by the time of the
‘Hiuen Tsang wrote an account about this arrival of the Maurayas.
place. Primarily it was a centre for learning
of Mahayana Buddhism. Harsha invited Which one of the following is correct in
monks from here to take part in respect of the above statements?
philosophical assembly at Kannauj.’
a) Both Statement-I and Statement-II
Select the correct answer using the options are correct and Statement-II is the
given below correct explanation for Statement-I.
b) Both Statement-I and Statement-II
a) Takshashila University are correct and Statement-II is not
b) Vikramshila University the correct explanation for
c) Valabhi University Statement-I.
d) Nalanda University c) Statement-I is correct and but
Statement-II is incorrect.
Q.49) Consider the following d) Statement-I is incorrect but
Mahajanapadas: Statement-II is correct.
1. Vajji
2. Magadha Q.51) Greek historians have recorded the
3. Malla name ‘Sandrakottus’ or ‘Sandrakoptus’ for
4. Kuru who among the following?

a) Kautilya
Which of the above given Mahajanapadas
b) Nandas
are considered as republic/ non-
c) Chandragupta Maurya
Monarchial?
d) Ashoka
a) 1, 2 and 4 only
b) 1, 3 and 4 only Q.52) Consider the following statements
c) 2, 3 and 4 only regarding the ‘Uttarapatha and
d) 1, 2 and 3 only Dakshinapatha’.
.
IASBABA’S AIPTS 2024 - PRELIMS TEST 1 (ANCIENT HISTORY)

Statement I: Uttarapatha route stretches d) None


from North-west India to present day
Gujarat coast. Q.54) Which of the following statements
Statement II: Dakshinapatha route stretches with reference to the age of the Mauryan
from port city of Tamralipti to Arikamedu. Empire is/are correct?
Which one of the following is correct in
1. Registration of birth and death,
respect of the above statements?
regulation of weights and care of
a) Both Statement-I and Statement-II
foreigners were some of the
are correct and Statement-II is the
administrative functions of Mauryas.
correct explanation for Statement-I.
2. Mauryas also maintained a navy.
b) Both Statement-I and Statement-II
3. Peasants were provided with free
are correct and Statement-II is not
irrigation facilities by the state.
the correct explanation for
Statement-I. Choose the correct answer using the code
c) Statement-I is correct and but below:
Statement-II is incorrect.
d) Statement-I is incorrect but a) 1 and 2 only
Statement-II is incorrect. b) 1 and 3 only
c) 2 and 3 only
Q.53) Consider the following statements d) 1, 2 and 3
with respect to the scientific knowledge of
Ancient Indian History: Q.55) Which of the following statements
is/are correct with respect to Bindusara?
1. The early Indian scientists
unanimously believed that earth is 1. Bindusara is known to have made a
stationary and the sun revolves gift of elephants to Selukas,
around the earth. Alexander’s ambassador.
2. Indians were able to calculate the 2. He was called as Amitraghat.
value of Pi and were aware of 3. He was assisted by Kautilya in his
Pythagoras theorem. administration.
3. Baudhayan was first to calculated the
Select the correct answer using the code
value of Pi.
below:
How many of the statements given above
a) 1 and 3 only
is/are correct?
b) 2 only
a) Only one c) 2 and 3 only
b) Only Two d) 1, 2 and 3
c) All Three
.
IASBABA’S AIPTS 2024 - PRELIMS TEST 1 (ANCIENT HISTORY)

Q.56) Consider the following statements the correct explanation for


regarding the Gupta period: Statement-I.
c) Statement-I is correct and but
1. The religious rights of lower varnas
Statement-II is incorrect.
improved.
d) Statement-I is incorrect but
2. The woman of upper varnas had
Statement-II is correct.
more freedom compared to lower
varnas.
Q.58) Consider the following statements
3. The Chinese traveler Fahien
about state control in Mauryan Empire.
mentions that 'Chandalas' separated
from society were Sudras. Statement I: Mauryan administration was
characterized by a vast bureaucracy and a
How many of the statements given above well knitted spy system.
is/are correct? Statement II: Mauryan administration was
a) Only one highly decentralized in nature.
b) Only Two
Which one of the following is correct in
c) All Three
respect of the above statements?
d) None
a) Both Statement-I and Statement-II
Q.57) With reference to Megasthenes’s are correct and Statement-II is the
account of Mauryan Empire, consider the correct explanation for Statement-I.
following statements: b) Both Statement-I and Statement-II
are correct and Statement-II is not
Statement I: Slavery was a regular practice
the correct explanation for
particularly in the domestic sphere.
Statement-I.
Statement II: Systematic Road connectivity is
c) Statement-I is correct and but
found between various economic centers
Statement-II is incorrect.
from northwest to eastern India and also
d) Statement-I is incorrect but
along the foothills of Himalayas.
Statement-II is correct.
Which one of the following is correct in
respect of the above statements? Q.59) Consider the following statements:

a) Both Statement-I and Statement-II 1. During the Mauryan rule, political


are correct and Statement-II is the unity was achieved for the first time.
correct explanation for Statement-I. 2. The land revenue was fixed at one
b) Both Statement-I and Statement-II third of the produce.
are correct and Statement-II is not 3. The Census appears to be a
permanent institution in the
Mauryan administration.
.
IASBABA’S AIPTS 2024 - PRELIMS TEST 1 (ANCIENT HISTORY)

How many of the statements given above 3. One of the foremost literary activities
is/are correct? of the Gupta period is to be seen in
the final redaction of the
a) Only one
Mahabharata.
b) Only Two
c) All Three Which of the above given statements is/are
d) None correct?

Q.60) Which of the following are among the a) 1 and 3 only


causes of fall of the Mauryan Empire? b) 1 and 2 only
c) 2 and 3 only
1. The religious policy of Ashoka. d) 1, 2 and 3
2. Large donations made by the empire
for the expansion of Buddhism. Q.62) Which one of the following
3. Neglect of the Northwest frontier representations are found on the coins of
and the threat from Scythians. Samudragupta?

Which of the statements given above is/are 1. Holding a bow and arrow
correct? 2. Armed with an axe and sword
3. The tiger slayer type
a) 1 and 2 only
4. Shooting at a lion
b) 1 and 3 only
c) 2 and 3 only Which of the above given types is/are
d) 1, 2 and 3 correct?

Q.61) With reference literature during the a) 1, 2 and 3 only


Gupta's age, Consider the following b) 2, 3 and 4 only
statements: c) 1, 2 and 4 only
d) 1, 2, 3 and 4
1. It reached its perfection in both form
and content, and almost all the best Q.63) Consider the following pairs with
works in its different branches were respect Rulers and its dynasty's:
composed before the end of this Rulers Dynasty
glorious period. Vasudeva I He was the last of
2. Sanskrit now replaced the Prakrit not the Shunga
only in epigraphic, but even in the emperors
religious and philosophical literature Devabhuti He was the last great
Kushan emperor
of the heterodox sects like the Jainas
Pulumavi IV He was the last of
and Buddhists. the Shatavahana
emperors
.
IASBABA’S AIPTS 2024 - PRELIMS TEST 1 (ANCIENT HISTORY)

How many of the following pairs is/are controlled the area along the Ayodha
correctly matched? Region.

a) Only one pair Which of the above given statements is/are


b) Only two pairs incorrect?
c) Only three pairs
d) None of the pairs are correctly a) 1 and 2 only
matched b) 2 and 3 only
c) 1 and 3 only
Q.64) Consider the following events in the d) All of the above
history of India:
Q.66) Consider the following pairs:
1. Establishment of Kadamba dynasty
2. The Battle of Vatapi 1. Akshapataladhyaksha: Accountant
3. Senguttuvan was succeeded by his General who was in charge of the
brother Perunjeral two offices of Currency and
4. Establishment of Pala Dynasty Accounts.
2. Sitadhyaksha: Director of Agriculture
What is the correct chronological order of and in charge of the cultivation of
the above events, starting from earliest Crown lands
time? 3. Lavanadhyaksha: Superintendent of
Ports, he controlled traffic and
a) 2-3-1-4
transit by waterways, riverine,
b) 3-2-1-4
oceanic, coastal routes, etc.
c) 3-1-2-4
d) 1-3-2-4 Which of the above given pairs are correctly
matched?
Q.65) Consider the following statements
with respect to the Guptas in Ancient India: a) 1 and 2 only
b) 2 and 3 only
1. Chandragupta I has been identified c) 3 only
with the first Gupta king whose name d) 1, 2 and 3
is mentioned in the Allahabad pillar
inscription. Q.67) With reference to the status of women
2. The Gupta Empire was the during Maurya’s period, consider the
contemporary of the empires of the following statements:
Satavahanas and Kushanas.
3. The Allahabad pillar inscription 1. Women were appointed as
mentions that the early Guptas bodyguards to the kings.
.
IASBABA’S AIPTS 2024 - PRELIMS TEST 1 (ANCIENT HISTORY)

2. Widow remarriage and divorce were c) Indo-Scythians, Indo-Greeks, Indo-


allowed for women. Parthians, Kushans
3. Practice of polygamy increased. d) Indo-Greeks, Indo-Parthians, Indo-
Scythians,Kushans
Which of the above given statements is/are
correct? Q.70) Consider the following statements
a) 1 only with respect to the differences between
b) 2 and 3 only Indus Valley and Mesopotamian civilizations:
c) 1 and 2 only Statement I: Mesopotamians built temples
d) All of the above for worship while no special places of
worship have been found in Indus Valley.
Q.68) With reference to Pushyamitra
shunga, Consider the following statements: Statement II: Mesopotamians were ruled by
a priest King while theory of Kingship or a
1. He was the founder of Shunga ruler class has not been proved for Indus
dynasty. Valley.
2. He was a contemporary of the great
grammarian, Patanjali. Which one of the following is correct in
3. He is the hero of Kalidasa's respect of the above statements?
Malavikagnimitra.
a) Both Statement-I and Statement-II
Which of the above given statements is/are are correct and Statement-II is the
correct? correct explanation for Statement-I.
b) Both Statement-I and Statement-II
a) 1 and 2 only are correct and Statement-II is not
b) 2 and 3 only the correct explanation for
c) 1 only Statement-I.
d) 1, 2 and 3 c) Statement-I is correct and but
Statement-II is incorrect.
Q.69) Which of the following option gives d) Statement-I is incorrect but
the correct sequence of dynasties that came Statement-II is correct.
to India, starting with the first and ending
with the last? Q.71) In the context of Jainism and
a) Indo-Greeks, Indo-Parthians, Buddhism, how Jainism is different from
Kushans, Indo-Scythians Buddhism?
b) Indo-Greeks, Indo-Scythians Indo- 1. Jainism recognised the existence of
Parthians, Kushans God while Buddhism denied it.
.
IASBABA’S AIPTS 2024 - PRELIMS TEST 1 (ANCIENT HISTORY)

2. Jainism condemn the varna system 1. Satavahanas were brahmanas even


while Buddhism does not condemn though the rulers gave the Buddhist
it. monks tax-free lands and promoted
3. Jainism believed in transmigration of Buddhism.
soul i.e. reincarnation while 2. Hinayana form of Buddhism was
Buddhism does not believed in it. followed widely in the Satavahana
Empire.
How many of the statements given above 3. Satavahanas worshipped Vaishnava
is/are correct? Gods such as Krishna and Vasudeva.
a) Only one
How many of the statements given above
b) Only Two
is/are correct?
c) All Three
d) None a) Only one
b) Only Two
c) All Three
Q.72) Which of the following rock edict d) None
contains the summary as well as an
explanation of Ashoka’s dhamma? Q.75) With reference to Harshavardhana,
a) Major Rock Edict XIII consider the following statements:
b) Major Rock Edict XI 1. He was a greatest king of
c) Major Rock Edict V Pushyabhuti dynasty.
d) Major Rock Edict X 2. The feudal practice of granting land
to officers begun during his period.
Q.73) Which of the following Pillar 3. Caste system was prevalent among
Inscription was erected by Asoka to Hindus during his period.
commemorate the birthplace of the
Buddha? Which of the statements given above is/are
correct?
a) Nigali-Sagar
b) Rummindei a) 1 and 2 only
c) Louriya-Araraj b) 1 and 3 only
d) Maski c) 2 and 3 only
d) 1, 2 and 3
Q.74) With reference to the religious aspect
of ‘Satavahan Dynasty’, consider the Q.76) With reference to religious beliefs of
following statements: Harappan people, consider the following:

1. Nature worship
.
IASBABA’S AIPTS 2024 - PRELIMS TEST 1 (ANCIENT HISTORY)

2. Worship of Mother Goddess


3. Belief in life after death Q.79) With reference to the Nalanda and
4. Faith in magic charms and sacrifices. Taxila universities, consider the following
statements:
Which of the above given is/are correct?
1. Nalanda was a formal university in
a) 1, 2 and 3 only the modern sense of the word
b) 2, 3 and 4 only whereas Taxila functioned under
c) 1, 2 and 4 only more informal conditions.
d) All of the above 2. Nalanda and Taxila had almost all the
infrastructure namely, multiple
Q.77) Which of the following classrooms, extensive library,
Samudragupta’s inscriptions are considered accommodation for professors, etc.
as Royal Charter? 3. Both Taxila and Nalanda university
1. Allahabad Stone Pillar had disciplines of Astronomy,
2. Eran Stone Pillar Mathematics, Politics and Science.
3. Nalanda Copper Plate
Which of the above given statements is/are
4. Gaya Copper Plate
correct?
Select the correct code: a) 1, 2 and 3 only
a) 1 and 2 only b) 2 and 3 only
b) 3 and 4 only c) 1 only
c) 2 and 3 only d) All of the above
d) All of the above
Q.80) In Gupta’s period the terms ‘Bardhya’
Q.78) Consider the following pairs with and ‘Khaya’ are related to which of the
respect to classification of land during Gupta following?
Period: a) These were the names of the taxes
1. Kshetra: Jungle or forest land imposed by the Kings on Cultivators.
2. Khila: Waste land b) These were the types of crops
3. Aprahata: Cultivatable land growing in the Gupta’s period.
c) These were related to the Irrigation.
How many pairs is/are correctly matched? d) These were the posts given in
millitary administration during the
a) One pair only
Gupta’s period.
b) Two pairs only
c) Three pairs
d) None of the above
.
IASBABA’S AIPTS 2024 - PRELIMS TEST 1 (ANCIENT HISTORY)

Q.81) With reference ‘Prices and 4. With a view to popularize and


Measurements’ during the Gupta’s period, propagate the doctrines of
consider the following statements: Mahayana Buddhism, Harsha
arranged at Kanyakubja a great
Statement II: The prices of the goods were
assembly which was presided over by
fixed by the Kings.
Hiuen Tsang, in 643 CE.
Statement II: In this period units of
measurements varied from place to place. Which of the above given statements is/are
correct?
Which one of the following is correct in
respect of the above statements? a) 1, 2 and 4 only
b) 3 and 4 only
a) Both Statement-I and Statement-II
c) 1 and 3 only
are correct and Statement-II is the
d) 1, 2 and 3 only
correct explanation for Statement-I.
b) Both Statement-I and Statement-II
are correct and Statement-II is not
Q.83) The Greeks introduced an
the correct explanation for
administrative system called ‘Strategos’. It
Statement-I.
stands for-
c) Statement-I is correct and but
Statement-II is incorrect. a) Divine origin of Kingship
d) Statement-I is incorrect but b) The idea of son of the soil
Statement-II is correct. c) Military Governorship
d) Hereditary Dual rule
Q.82) With reference to the religious
practice during the Harshavardhana’s Q.84) Consider the following statements
period, consider the following statements: with respect ‘Art and Architecture of the
Chalukya’s of Badami’:
1. Brahmanism becomes strengthened
during this period. 1. They developed the Deccan or
2. Decline of Buddhism despite the Vesara style in the building of
patronage given to it by Harsha but structural temples.
Jainism did not undergo any major 2. They perfected the art of stone
changes and it made neither building, that is, the stones finely
progress nor any decay. joined without mortar.
3. Vaishnavism became the main 3. Under their auspices, the Buddhists,
theistic of this period whereas Jainas and Brahmins were equally
Shaivism was gradually declining preferred in building cave temples.
during this period.
.
IASBABA’S AIPTS 2024 - PRELIMS TEST 1 (ANCIENT HISTORY)

Which of the above given statements is/are defeated and killed by


correct? Mahendravarman's son and
successor, Narasimhavarman II.
a) 1 and 3 only
b) 2 and 3 only Which of the above given statements is/are
c) 1 and 2 only correct?
d) 1, 2 and 3
a) 1, 3 and 5 only
Q.85) Which of the following ruler have got b) 2 and 4 only
the title of ‘Vatapikonda’? c) 1, 3 and 4 only
d) All of the above
a) Pulikeshi II
b) Mahendravarman I Q.88) With reference to the temples of
c) Narasimhavarman I Badami Chalukya’s, which of the following
d) Mangalesha temples are situated in Pattadakal:

Q.86) ‘Ghatiyantra’ which was used in early 1. Sangamesvara Temple


India refers to or used for- 2. Virupaksha Temple
3. Ladkan Temple
a) Manufacture of vessels of metals 4. Jambulinga Temple
b) Pouring of water in sacrificial rituals 5. Mallikarjuna Temple
c) The metal plate which used in
conducting tantric Rituals Select the correct code:
d) Irrigation of water from well
a) 1, 2, 3 and 4 only
b) 1, 2, 4 and 5 only
Q.87) Consider the following statements
c) 2, 3, 4 and 5 only
with respect to Pulakesin II:
d) 1, 2, 3, 4, and 5
1. He is the founder of Chalukya
dynasty. Q.89) Match the following Gupta kings with
2. He sent an embassy to the Persian their inscriptions:
king Khusrau II.
Kings Inscriptions
3. He was defeated by Harshavardhana Samudragupta Menrauli Iron Pillar
on the banks of Narmada. Chandragupta II Eran Stone Pillar
4. He also took on the title Skandagupta Paharpur Copper Plate
‘Daskshinapatheshwara’, similar to Inscription
Harsha's Uttar Natheshwara after Budhagupta Bhitari Pillar
defeating Harshavardhana. Inscription
5. He defeated Pallava king
Mahendravarman I, but was
.
IASBABA’S AIPTS 2024 - PRELIMS TEST 1 (ANCIENT HISTORY)

Select the correct answer from the codes c) 1 and 3 only


below: d) 1, 2 and 3

a) 1-B, 2-A, 3-D, 4-C


b) 1-B, 2-A, 3-C, 4-D
Q.92) With reference to the Kushana’s,
c) 1-A, 2-B, 3-D, 4-C
consider the following statements:
d) 1-A, 2-B, 3-C, 4-D
1. Kujula Kadphises was the first Yuezhi
Q.90) Which of the following taxes are the chief to lay the foundation of the
agrarian taxes during the Gupta’s period? Kushana Empire in India.
2. The age of Kanishka witnessed the
1. Bhaga
execution of the best work in
2. Bhoga
Gandhara style.
3. Kara
3. Vima Kadphises was the Kushan
4. Shulka
emperor to first introduce gold
Select the correct answer from the codes coinage, in addition to the existing
below copper and silver coinage.

a) 1, 2 and 3 only Which of the above given statements is/are


b) 2, 3 and 4 only correct?
c) 1, 3 and 4 only
a) 1 and 3 only
d) 1, 2, 3 and 4
b) 2 and 3 only
c) 1 and 2 only
Q.91) With reference to the Kadmba
d) 1, 2 and 3
dynasty, consider the following statements:

1. Mayurasharma was the founder of Q.93) Consider the following statements:


the dynasty.
1. Sittanavasal Temple-cave was
2. Talagunda pillar inscription is major
originally attributed to Pallava King
evidence which gives information
Mahendravarman I
about their origin.
2. Sittanavasal caves are related to
3. Mayurasharman was succeeded by
Buddhism.
Kakusthavarman, was the greatest
3. Sittanavasal is the name used
king among the kadamba rulers.
synonymously for the hamlet and the
Which of the above given statements is/are hillock that houses the Arivar Kovil
correct? i.e., temple of Arihats – Buddhist
Monks who conquered their senses
a) 1 and 2 only 4. This is the only place in Tamil Nadu
b) 2 and 3 only where one can see Pandya paintings.
.
IASBABA’S AIPTS 2024 - PRELIMS TEST 1 (ANCIENT HISTORY)

Which of the following statement(s) is/are a) 1 and 2 only


correct? b) 2 and 3 only
c) 1 and 3 only
a) 1 and 4 only
d) 1, 2 and 3
b) 2 and 3 only
c) 1, 2 and 4 only
Q.96) Which of the following provides
d) All of the above
information about South Indian Kingdoms?

Q.94) Which of the following ports are a) Indica of Megasthenes


mentioned in 'Periplus', which were b) Rock Edict II of Ashoka
important in the trade between India and c) Hathi-gumpha inscription
Roman empire during the Sangam Age? d) All of the above

1. Naura
Q.97) Match the following Lands with its
2. Tyndis
Regions in Sangam age:
3. Muziris
4. Nelcynda

Select the correct answer using the code Name of the land Regions
1. Kurinji A. Pains
below:
2. Palai B. Jungles
a) 1, 2 and 3 only 3. Mullai C. Dry Lands
b) 2, 3 and 4 only 4. Marudam D. Hills
5. Neydal E. Coast
c) 1, 2 and 4 only
d) 1, 2, 3 and 4 Select the correct answer from the codes
below:
Q.95) In the context of Silappadigaram,
consider the following statements: a) 1-B, 2-A, 3-D, 4-C, 5-E
b) 1-B, 2-A, 3-C, 4-D, 5-E
1. It was the earliest and greatest of the c) 1-A, 2-B, 3-D, 4-C, 5-E
epics of the Sangam Age written by d) 1-D, 2-C, 3-B, 4-A, 5-E
Illango Vadigal.
2. It was the story of Senguttuvan early Q.98) Consider the following pairs with
life and conquest policies. respect minor classes and its occupations
3. It marks the beginning of Kannagi during the Sangam age:
cult or pattini cult, that is worship of
Kannagi as the ideal wife. Minor classes Occupation
1. Kadaisiyar Agricultural
Which of the above given statements is/are labourers
correct? 2. Eyinar Hunters
3. Pulaiyans Rope-makers
.
IASBABA’S AIPTS 2024 - PRELIMS TEST 1 (ANCIENT HISTORY)

How many of the following pairs is/are a) Chief of the villages


correctly matched? b) Land tax collector
c) Revenue accountant
a) Only one pair
d) Officer who maintained foreign trade
b) Only two pairs
c) Only three pairs
d) None of the pairs are correctly
matched

Q.99) With reference to the social conditions


during the Sangam Age, consider the
following statements:

1. Poetry, music and dancing were


quite popular among the Sangam
people and the poets were men and
women drawn from all classes of
society.
2. Avvaiyar and Nachchellaiyar were
only two women who immensely
contributed to Sangam literature.
3. The position of women suffered and
remarriage of widows was generally
not accepted.
4. For Burial practices no single method
was adopted for the disposal of the
dead, either cremation or
inhumation was followed.

Which of the above given statements is/are


correct?

a) 1 and 2 only
b) 2 and 3 only
c) 3 and 4 only
d) 1, 3 and 4 only

Q.100) The office of ‘Ayakkarar’ in Sangam


period refers to?
.
IASBABA’S AIPTS 2024 – PRELIMS TEST 1 (ANCIENT HISTORY) SOLUTIONS

Q.1) Solution (c)

Explanation:

● Palaeolithic culture of India developed in the Pleistocene period. The Upper Palaeolithic
age coincided with the last phase of the Ice Age. Hence statement 1 is correct)
● Palaeolithic sites are found in practically all parts of the country except the alluvial plains
of the Indus and Ganges. (Hence statement 2 is incorrect)

Important Facts:

• The old stone age or palaeolithic age in India is divided into three phases according to the
nature of the stone tools used by the people and also according to the nature of the
change of climate.
• Lower Palaeolithic Age: up to 100,000 BC, Tools-Handaxe, chopper, cleavers, chopping
tools.
• Middle Palaeolithic Age: 100,000 BC – 40,000 BC, tools- Flake tools
• Upper Palaeolithic Age: 40,000 BC – 10,000 BC, tools- Burns and Burins.

Q.2) Solution (d)

Explanation:

● The domestication of animals started in the later Mesolithic period. The Neolithic people
were acquainted to agriculture. Earliest Evidence of domestication of animals (Sheep,
dogs, cattle and goat) were found in Bagor and Adamgarh around 5000 BCE. (Hence
statement 1 is incorrect)
● Burzahom is an Important Neolithic site 16 km near to Kashmir, While Mehrgarh is
neolithic site where the first known farming community developed is located in
Balochistan, Pakistan. (Hence statement 2 is correct)

Q.3) Solution (c)

Explanation:

● Early Harappan Phase: Also known as the Regionalization Era. It was the formative,
proto-urban phase of the Harappan Civilisation. (Hence statement 1 is correct)
● Mature Harappan Phase: Also known as the Integration Era. It was the urban phase of
the Harappan civilizations, which shows the characteristics of a full-fledged Civilisation. It
.
IASBABA’S AIPTS 2024 – PRELIMS TEST 1 (ANCIENT HISTORY) SOLUTIONS

is pertinent to note that the unqualified term, ‘Harappan Culture’ or ‘Harappan


Civilisation’, primarily refers to this mature urban phase. (Hence statement 2 is correct)
● Late Harappan Phase: The last phase of the Harappan Civilisation, which is also known as
the Localisation Era, was the post-urban phase defined by a decline in the cities. This is
the final period in the Harappan civilisation and refers to the fragmentation of the culture
of the Integration Era. (Hence statement 3 is correct)

Q.4) Solution (c)

Explanation:

● Mesolithic Age was an ancient cultural stage that existed between the Paleolithic Period
with its chipped stone tools, and the Neolithic Period with its polished stone tools. It is
also called Microlithic age as the tools used were chipped stone tools also known as
microliths. (Hence statement 1 is correct)
● The Mesolithic people lived on hunting, fishing and food gathering. At a later stage, they
also domesticated animals. (Hence statement 2 is correct)
● The excavated sites in Uttar Pradesh like Chopani Mando, Sarai Nahar Rai, Mahadaha and
Damdama contribute to the study of archaeology of Mesolithic period. In an excavation
1978-79, thirty-five well-preserved fossil hominid skeletons were discovered. (Hence
statement 3 is correct).

Q.5) Solution (a)

Explanation:

Neolithic, also called New Stone Age, final stage of cultural evolution or technological
development among prehistoric humans. It was characterized by

● Stone tools shaped by polishing and grinding. (Hence statement 1 is correct)


● Dependence on domesticated plants or animals.
● Settlement in permanent villages.
● Appearance of carved objects from wood, pottery and weaving. (Hence statement 2 is
correct)
● Built shelter and tombs.
● Neolithic followed by the Paleolithic Period, or age of chipped-stone tools, and preceded
the Bronze Age, or early period of metal tools. Iron was not known in this period. (Hence
statement 3 is incorrect)
.
IASBABA’S AIPTS 2024 – PRELIMS TEST 1 (ANCIENT HISTORY) SOLUTIONS

Q.6) Solution (d)

Explanation:

● Towns of Harappa and Mohenjodaro were divided into parts namely citadel and lower
town. Citadels were occupied by members of the ruling class and the lower town was
inhabited by the common people. (Hence statement 1 is correct)
● Practically every house had a bathroom which, was on the side of the street for the
convenient disposal of water. In the number of cases, latrines have been found which
were placed between the bathroom and the street wall. The bathrooms and latrines were
sometimes found on the ground floor and sometimes on the first floor. There was an
arrangement of wastewater to run through a brick channel in the thickness of the wall,
which was received below in a cesspit. Pottery pipes were used for the drainage system.
These were fitted both vertical and horizontally. They were so laid that the end of the
fitted into socket of another. A brick-lined channel flowed down every street, sometimes
even in small lanes, into the tributary drains from the houses on either side. (Hence
statement 2 is correct)

● No Citadel has been found in Chanudaro. So, statement 3 is incorrect.

● Dholavira was divided into three divisions, The upper, middle and lower town. Hence
statement 4 is correct.

Q.7) Solution (d)

Explanation:

Mohenjodaro is one of the largest of the Indus Valley Civilization sites.

● During the archaeological investigations, archaeologists discovered several human bones


and other skeletal remains on this mound. To refer to it as the "Mound of Dead," they
gave it the names ‘MOHEN’ which means Mound, and ‘JODARO’ which means of the
Dead. (Hence statement 1 is correct)
● It is located on the right (west) bank of the lower Indus River in Larkana District of Sindh,
Pakistan. (Hence statement 2 is correct)
● Important Findings:
o Great Bath: The most famous building found at Mohenjo-Daro is a great bath. It is
a 6×12-meter specimen of beautiful brick work. It has a tank with 11.88×7.01
lengths and breadths and 2.43 meters depth along with steps on the north and
south sides.
.
IASBABA’S AIPTS 2024 – PRELIMS TEST 1 (ANCIENT HISTORY) SOLUTIONS

o Granary: A granary has been found which the largest building of the Mohenjo-
Daro is. This granary is divided into 27 rooms of different size and shape.
o Assembly Hall: A square pillared hall with 90X90 ft is another important building
found at Mohenjo-Daro. The scholars agree that this pillared hall was a site for
social gatherings.
o Seated male sculpture or "Priest-King" from Mohenjo-Daro is made of steatite. It
has the following other features: The fillet headband with circular inlay ornament
on the forehead and on the right upper arm. The hair is carefully combed towards
the back of the head and absence of bun. (Hence statement 3 is correct)
● Along with the major structures, from Mohenjo-Daro some important Craft also
discovered. Some of the important discoveries are the statues. Like the dancing girl
statue, It was a bronze-made structure. The Priest Statue, it was made with Soapstone.
Though historians are not sure whether this priest was ruled on Mohenjo-Daro or not.
But still, it is called the statue of “Priest King”. In this statue, a breaded man image is
displayed. (Hence statement 4 is correct)

Q.8) Solution (c)

Explanation:

The archaeological site of Kalibangan is an ancient Indus Valley civilization site in Rajasthan has
discovered evidence of the world's earliest ploughed agricultural field ever revealed through an
excavation by Dr. B Lal. It is also a site which has given evidence of earliest recorded Earthquake.
The earthquake is dated back to 2600 BC. A wooden furrow and seven fire altars in a row have
been discovered at Kalibangan reveal that the people were ritualistic and believed in worship of
fire and implying the practice of sacrifice. (Hence option (c) is correct answer)

Q.9) Solution (c)

Explanation:

The urban settlement in Harappa is generally divided into two sections, one smaller but higher
and the other much larger but lower. Archaeologists designate these as the Citadel and the Lower
Town respectively. The Citadel owes its height to the fact that buildings were constructed on mud
brick platforms. It was walled, which meant that it was physically separated from the Lower
Town. (Hence option (c) is correct answer)
.
IASBABA’S AIPTS 2024 – PRELIMS TEST 1 (ANCIENT HISTORY) SOLUTIONS

Q.10) Solution (b)

Explanation:

Lost-wax technique is also called cire-perdue, it is a method of metal casting in which a molten
metal is poured into a mold that has been created by means of a wax model. Once the mold is
made, the wax model is melted and drained away. A hollow core is filled with molten metal which
is solidified to get the casting. (Hence option (b) is correct answer)

Q.11) Solution (a)

Explanation:

● Walls made of burnt bricks were raised for protection, which shows that floods took place
annually. massive mud-brick platforms were erected and faced with fired brick,
apparently with the objective of raising the level of the city safely above lake waters. One
such embankment, partially excavated by the expedition is some 70 feet wide and well
over 25 feet high (Hence statement 1 is correct)
● The humped bulls were favored by the Harappans and from the very beginning dogs were
regarded as pets. (Hence statement 2 is incorrect)

● They relied heavily on the flooding of the Indus River for crop irrigation and agriculture,
and they built wells, drains, channels and dams to control the river water. Hence
statement 3 is incorrect.

Q.12) Solution (a)

Explanation:

● The Harappan Civilization's seals were one of their most remarkable and artistic creations.
Since 1921, approximately 3500 seals have been discovered.
● The majority of these seals were made of Steatite, a relatively soft stone found in river
beds. Aside from that, copper, terracotta, chert, faience, and agate were used to make
those seals. Archaeologists have also discovered gold and ivory seals from the Harappa
sites in recent years. (Hence statement 1 is correct)
● Some Harappan civilization seals were discovered on the dead bodies. These seals have a
threaded hole, implying that they were used as amulets or as a necklace. (Hence
statement 2 is correct)
.
IASBABA’S AIPTS 2024 – PRELIMS TEST 1 (ANCIENT HISTORY) SOLUTIONS

● Animal impressions were carved on the surfaces of the seals on the other sides. Tiger,
buffalo, elephant, rhinoceros, bison, goat, unicorn, ibex, humped bull, crocodile, and
other animal motifs were used. (Hence statement 3 is correct)
● Several Harappan seals have also been discovered in Mesopotamia, Central Asia, and
along the Arabian Peninsula's coasts. Similarly, Mesopotamia seals were discovered in
Lothal. This confirms that seals were used in trading and that trade took place between
the two world's great ancient civilizations (Harappan and Mesopotamia civilizations).
(Hence statement 4 is incorrect)

Q.13) Solution (a)

Explanation:

● Sutkagendor or Sutkagan Dor is located on western side of river Dasht, its confluence with
a smaller stream, called as the Gajo Kaur. It is close to Iranian broader, in Pakistan
Baluchistan Providence. It was a smaller settlement of Harappan Civilization with
substantial stone, walls and gateways. It was first discovered by Major Edwar Mockler in
1875, a small-scale excavation was conducted at this time. Further excavations were done
by Aurel Stein in 1928 and more extensively by Geroge F. Dales in 1960. (Hence option a)
is correct answer)
● Surkotada is situated in Ropar Taluka of Kutch district of Gujarat. This was a small
settlement of Harappan Civilization. The mound from here was discovered by J. P. Joshi
in 1964, under the Archaeological Survey of India.
● Kotla Nihangkhan is located in southeast part of Ropar city of Punjab. It was a major
archaeological site of Bronze Age Harappan Civilization from 3300-1300 BCE.
● Aalamgirpur is located along Yamuna River from the Harappan-Bara period, from Meerut,
district, Uttar Pradesh. This was excavated under Archaeological Survey of India from
1958. This is also called as 'Parasaram-ka-Khera'.

Q.14) Solution (d)

Explanation:

Harappan sites River


1 Lothal Bhogava
2 Kalibangan Ghaggar
3 Chanhu-Daro Indus
.
IASBABA’S AIPTS 2024 – PRELIMS TEST 1 (ANCIENT HISTORY) SOLUTIONS

4 Surkotada Sabarmati
5 Ropar Sutlej

Q.15) Solution (c)

Explanation:

● Many scholars identify him with God Pashupati (Lord of beasts) or ‘ProtoShiva’ though
some dispute it. In another instance, a deity is shown with horns and flowing hair standing
nude between the branches of a Pipal tree and a worshipper is kneeling in front. It may
represent tree worship. Animal worship also appears to be popular among the
Harappans. Thus, they may be called animist because in animism there is a belief that
all natural things, such as plants, animals, rocks, and thunder, have spirits (Hence
statement 1 is correct)
● At Kalibangan and Lothal, Fire altars were found in forts and houses. There were platforms
in residences which used to support the fire altars. The altars were arranged in a row and
there were stairs to access those fire altars. (Hence statement 2 is correct)
● These people had a belief in life after death. The dead bodies, therefore, were either
buried or after the cremation, their remains were preserved in a jar. With the dead
bodies, the Harappan people also buried animals, birds, fish, beads, bangles, etc, so that
the dead person might use them in need. (Hence statement 3 is correct)

Q.16) Solution (b)

Explanation:

● The Harappan civilization was mainly urban-based.


● A piece of woven cotton has been recovered from Mohenjodaro which was evidence for
the cotton cultivation in the Indus Valley.
● The inhabitants of Harappa had knowledge of both copper and bronze. But they were
most probably unaware of the use of iron.
● The standard Harappan seals were made of Steatite.

(Hence option (b) is correct answer)

Q.17) Solution (c)


.
IASBABA’S AIPTS 2024 – PRELIMS TEST 1 (ANCIENT HISTORY) SOLUTIONS

Explanation:

● Neolithic Age (10000-4500 BCE): It is the first reference of pottery in this period. It is
hand-made pottery, but later periods also utilized a foot-wheel.
● Chalcolithic Age (4500- 2000 BCE): The prevalence of various ceramic cultures
distinguishes it. Black and red-ware pottery, black-on-red ware, and ochre-colored
pottery are other examples.
● Indus Valley Civilization (3300 BCE- 1500 BCE): Polished Ware was one of the pottery
traditions that existed during the period. Pottery with a rough surface, Harappa Burial
Pottery, Ochre colored pottery (OCP), Black-grey burnished ware, Black-on-red ware,
Grey-ware, and painted grey-ware are all examples of pottery with a rough surface.
● Vedic Age (1500- 500 BCE): Painted Grey-ware (PGW), Northern Black Polished ware
(NBPW), and megalithic pottery unearthed in Kerala are examples of pottery traditions
that existed during the period.
● Mauryan Period (321 BCE- 185 BCE): The pottery wheel became widely used. There are
numerous different types of ceramics linked with the Mauryan period. However, the most
advanced technique may be observed in a form of pottery called Northern Black Polished
Ware (NBP) was popular during the previous and early Mauryan eras.

(Hence option c) is the correct answer)

Q.18) Solution (d)

Explanation:

● The Battle of Ten Kings was fought between the Bharatas on one hand and the host of
ten kings on the other, on the bank of river Parushni, identical with the river Ravi. It gave
victory to Sudas, leader of Bharata tribe and established the supremacy of Bharatas.
(Hence statement 1 is correct)
● The Purus, Kurus and Tritsu were tribal ruling clans among Aryans. (Hence statement 2 is
correct)
● The archaeological evidences shows that they possessed horses, chariots and some better
arms made of Bronze. They owed their success in India to such possessions only. When
they settled in western part of the subcontinent, they possibly used copper, supplied by
the Khetri mines in Rajasthan. (Hence statement 3 is correct)

Q.19) Solution (b)


.
IASBABA’S AIPTS 2024 – PRELIMS TEST 1 (ANCIENT HISTORY) SOLUTIONS

Explanation:

Purana a kind of didactic literature comprising history, anecdotes, and legends. In contrast to the
vedas, the Puranas are written in easy language and in the form of stories so that people can
derive enjoyment from them as well as learn about religious matters.

Matsya Purana, list "5 characteristics" or "five signs" of a Purana. These are called the Pancha
Lakshana and are topics covered by a Purana were following:

● Sarga: describes why and how the universe was created. (Hence pair 1 is correctly
matched)
● Pratisarga: is the creation of a new universe after the destruction of the old one. Every
creation ends in destruction, and every destruction is followed by a new creation. The
God of every new creation is Brahma. (Hence pair 2 is incorrectly matched)
● Vamsa: Genealogy of the sages, Gods, and kings.
● Manvantara: It is a cosmic cyclic period of time identifying the duration or era, reign, or
age of a Manu, the progenitor of mankind
● Vamsanucharita: Text relating rise and fall of kings belonging to royal dynasties, both the
Suryavanshis who claimed their descent from the Sun and Chandravanshis who claimed
their descent from the Moon. (Hence pair 3 is correctly matched)

Q.20) Solution (b)

Explanation:

In Early Vedic Period the basic unit of political organization was kula or family. Several families
joined together on the basis of their kinship to form a village or Grama. The leader of Grama was
known as Gramani. A group of villages constituted a larger unit called Visu. It was headed by
vishayapati. (Hence option b) is correct answer)

Q.21) Solution (b)

Explanation:

● Sruti is solely of divine origin and contains no specific concepts of law. Text contained in
Shruti is what has been revealed by God himself. Thus, it is not open to judgment or
evaluation. Sacred text that is called Shruti is unquestionable and canonical in nature.
(Hence statement 1 is correct)
.
IASBABA’S AIPTS 2024 – PRELIMS TEST 1 (ANCIENT HISTORY) SOLUTIONS

● Smriti comprises that part of Vedic literature that is remembered. These texts are less
authoritative than Vedic Shruti literature, but they are used to interpret and understand
Vedic philosophy. These texts are also regarded as a "friendly treatise" for early Vedic
Literature. (Hence statement 2 is correct)

Q.22) Solution (b)

Explanation:

The Kassite inscription of 1600BC and Mitanni inscriptions of 14 thcentury BC found in Iraq,
mentions about Rigvedic Aryans. These inscriptions provide some information about the Aryans
suggesting that a branch of Aryans moved towards the west from Iran. Hence Statement 1 is
incorrect.

Hittites, Mitannians and the ruling class of the Kassites belonged to a very large ethno-linguistic
group called 'Indo-European', Hence statement 2 is correct.

(Hence option (b) is correct answer)

Q.23) Solution (b)

Explanation:

The Upanishad are the Vedic literature comprises the philosophy of Vedic sages about the nature
of the ultimate reality, the process of creation the nature of the self and its relation with the
ultimate reality, the greatest value of life and the right way of living.

It appears today general agreement among all the Upanishads on the following point:

● Aatma (self) is the ultimate enduring and unitary reality in the individual.
● Brahma is the ultimate reality in and beyond the objective world.
● The pure Atma and Brahma are one. An individual would be right in saying ‘I am Brahma.’
● The one Brahma appears as many because of Maya, the wonderful power of Brahma.
● The union of Aatma with Brahma is called ‘Moksha’. It is a state that liberates one from
the chain of birth and death and is the highest goal of human life.
● All unhappiness and sufferings of man or due to ignorance of the fact that man is the
Atman, which is the same as the Brahma.
● To realise the Atman, one has to give all the Desire wordly and finite objects, purify one’s
intellect, and live a righteous life.
.
IASBABA’S AIPTS 2024 – PRELIMS TEST 1 (ANCIENT HISTORY) SOLUTIONS

(Hence option b) is correct answer)

Q.24) Solution (c)

Explanation:

● In the early Vedic age women enjoyed an honored place in the society. The wife was the
mistress of the household and authority over the slaves. In all religious ceremonies she
participated with her husband. Some of the rituals have cared for incomplete without the
participation of the wife.
● The education of women was not neglected. They received education along with their
male counterparts. Some of the women used to pursue education for a whole life. They
were known as “Brahmavadini”. Women like Apala, Ghosa, and Lopamudra & Vishwavara
were so highly educated that they composed Hymns of Rig-Veda and attained the rank of
Rishis. (Hence statement 1 is correct)
● They also participated in Sabha & Samiti.
● Prada system was not prevalent in the society. Sati system was also not prevalent in the
Vedic society. (Hence statement 2 is incorrect)
● The girls were married only after attaining puberty. Child marriage was not prevalent in
the society. (Hence statement 3 is correct)
● The practice of ‘Swayamvara’ was also prevalent in the society.
● Monogamy was the general Practice. Polygamy was, of course, practiced and it was
confined only to Rings and chiefs. Remarriage of widows was permitted.
● The women were not independent persons in the eye of the law. They had to remain
under the protecting care of their male relations. (Hence statement 4 is correct)
● The profession like spinning/weaving was limited to women.

Q.25) Solution (b)

Explanation:

The Maharshi Badrayan Vyas Samman, named after Sage Veda Vyas, the illustrated author of
Mahabharata, is a presidential award instituted in the year 2002, to honour young Indian scholars
in their recognition of their significant contribution to classical Indian languages.

● He is considered as the author of Brahma Sutras, which are also known as Vedanta Sutras.
(Hence statement 1 is correct)
.
IASBABA’S AIPTS 2024 – PRELIMS TEST 1 (ANCIENT HISTORY) SOLUTIONS

● He is also regarded as the founder of the Vedanta system of philosophy. (Hence


statement 2 is correct)

More about Badrayan Vyas Samman:

• The awards of Certificate of Honour and Maharshi Badrayan Vyas Samman are Indian
Presidential honors which are conferred on academics by the President of India once a
year on the Indian Independence Day, celebrated on 15 August; in recognition of their
substantial contribution in the various fields of languages including Arabic, Kannada,
Sanskrit, Malayalam, Oriya, Pali, Persian, Prakrit and the Telugu language.
• The awards come under the umbrella of the language division of the Ministry of
Education's Department of Higher Education. The award takes its name from Bādarāyaṇa,
the founder of the Vedanta system of Philosophy, who wrote the Vedāntasūtra k.a.
Brahmasūtra.

Q.26) Solution (c)

Explanation:

● Vidatha was tribal assembly, they exercised deliberative, military and religious functions.
The Vidatha seems to be a more popular assembly than either Sabha or Samiti in the Rig
Vedic period. In later Vedic times popular assemblies lost importance and royal power
increased at their cost. The Vidatha completely disappeared. (Hence statement 1 is
correct)
● The Sabha became more important than the Samiti. They came to be dominated by the
chiefs and the rich nobles. Women were not allowed to attend the Sabha which was now
dominated by the nobles and the Brahmana’s. (Hence statement 2 is incorrect)

Q.27) Solution (d)

Explanation:

● Bhagadugha: An officer responsible to collect taxes.


● Gramani: Head of the village.
● Madhyamasi: Mediator of Disputes.
● Akshavapa: The officer to maintain accounts.

(Hence option (d) is correct answer)


.
IASBABA’S AIPTS 2024 – PRELIMS TEST 1 (ANCIENT HISTORY) SOLUTIONS

Q.28) Solution (d)

Explanation:

● Lomas Rishi cave in Barabar hills belongs to the Mauryan period. The cave was patronised
by Ashoka for the Ajivika sect. (Hence statement 1 is correct)
● The shaft of the Ashokan pillars was monolithic unlike that of Achaemenian pillars. They
were inscribed from a single piece of stone. Mainly chunar sandstones were used for this
purpose. (Hence statement 2 is incorrect)
● Monumental images of Yaksha and Yakshini were quite popular during this period. (Hence
statement 3 is correct)

Q.29) Solution (d)

Explanation:

● Initially, only men were allowed into the sangha, but later women also came to be
admitted. According to Buddhist texts, this was made possible through the mediation of
Ananda, one of the Buddha’s dearest disciples, who persuaded him to allow women into
the sangha. The Buddha’s foster mother, Mahapajapati Gotami was the first woman to
be ordained as a bhikkhuni. Many women who entered the sangha became teachers of
dhamma and went on to become theris, or respected women who had attained
liberation. (Hence statement 1 is incorrect)
● The Buddha regarded the social world as the creation of humans rather than of divine
origin. Therefore, he advised kings and Gahapatis to be humane and ethical. Individual
effort was expected to transform social relations. (Hence statement 2 is correct)

Q.30) Solution (b)

Explanation:

● Nidanakatha is the first connected life story of Buddha. It is a non-canonical ancient


Buddhist text, ‘The Introduction to the Jataka’, that gives a chronological account of
Shakyamuni Buddha’s life and is the source of many myths about the Buddha. (Hence
statement 1 is correct)
.
IASBABA’S AIPTS 2024 – PRELIMS TEST 1 (ANCIENT HISTORY) SOLUTIONS

● Apart from texts in Pali, there are several Buddhist works in Sanskrit and in a mixture of
Prakrit and Sankrit. The trend towards the use of Sanskrit in Buddhist literature got
intensified in the Mahayana schools. (Hence statement 2 is incorrect)
● Lalitavistara is a hagiography of Buddha which is associated with the Sarvastivada School,
the canon of which is written in Sanskrit. But Lalitavistara is also tinged with Mahayana
elements that is using Sanskrit and mixed Prakrit- Sanskrit. (Hence statement 3 is correct)

● The Dipavamsa & the Mahavamsa (both in Pali) – both give historical and mythical
accounts of the Buddha’s life, Buddhist Councils, Asoka and the arrival of Buddhism to
Sri Lanka.

● Visuddhimagga (the path to purification written by Buddhaghosa) – deals with the


development from the purity of discipline to enlightenment (Nibbana). (Hence statement
4 is incorrect).

Important Points:

• The early Buddhist literature is divided into canonical and non-canonical texts:
• Canonical texts: are believed to be the actual words of the Buddha. Canonical texts are
books which lay down the basic tenets and principles of Buddhism such as the Tipitakas.
• Non-canonical texts or semi-canonical texts: these are commentaries and observations
on canonical texts, quotes, definitions, historical information, grammars and other
writings in Pali, Tibetian, Chinese and other East Asian languages. Some important ones
are:
• Mahavastu (written in Sanskrit-Prakrit mixed) – it is about the sacred biography, i.e
hagiography of the Buddha.
• Nidanakatha – first connected life story of Buddha.
• The Dipavamsa & the Mahavamsa (both in Pali) – both give historical and mythical
accounts of the Buddha’s life, Buddhist Councils, Asoka and the arrival of Buddhism to Sri
Lanka.
• Visuddhimagga (the path to purification written by Buddhaghosa) – deals with the
development from the purity of discipline to enlightenment (Nibbana).
• Milindapanho (in Pali) – consists of a dialogue between the Indo-Greek king
Milinda/Menander and the monk Nagasena on various philosophical issues.
• Nettipakarana (The book of guidance) – which gives a connected account of the Buddha’s
teachings.

Q.31) Solution (b)


.
IASBABA’S AIPTS 2024 – PRELIMS TEST 1 (ANCIENT HISTORY) SOLUTIONS

Explanation:

● Gandhara School flourished in two stages in the period from 50BC to 500AD. While the
former period was known for the use of bluish-grey sandstone, the later phase used mud
and stucco for making sculptures. (Hence statement 1 is correct)
● Kushan rulers especially Kanishka extended his wholehearted patronage to Buddhism. He
also held a Buddhist council in Kashmir when doctrine of Mahayana Buddhism was
finalised. The Kushan Empire brought together masons and other artisans trained in
different schools and countries. This gave rise to several schools of art: Central Asian,
Gandhara, and Mathura. (Hence statement 2 is correct)
● In Gandhara School, Buddha is shown in a spiritual state, seated in position of a yogi.
Buddha is shown in delighted mood in Mathura school of art. (Hence statement 3 is
incorrect)
● Protuberance is shown around the head of Buddha signifying the omniscience of Buddha.
(Hence statement 4 is correct)

Important Points:

• Gandhara art flourished in India during the reign of the Kushan Empire.
• Kanishka, the greatest of the Kushanas, was a well-known patron of the arts and building.
During his reign, the Gandhara School of painting flourished.
• Greek techniques had a big effect on Gandhara School.
• The Greek invaders brought the traditions of Greek and Roman sculptors with them,
which affected the region's native traditions.
• As a result, the Gandhara School became known as the Greco-Indian School of Art.
• Gandhara school is also known as Indo- Greek art because of the heavy influence of Greek
or Hellenistic sculpture.
• The Greek invaders brought the traditions of Greek and Roman sculptors with them,
which affected the region's native traditions.
• While Mathura School was created by indigenous peoples without the influence of
outside cultures.
• The narrative and imagery of all three religions of the time, Buddhism, Hinduism, and
Jainism were impacted by the Mathura School's sculptures.
• The images were based on Yaksha images discovered during the Mauryan period.

Q.32) Solution (c)

Explanation:
.
IASBABA’S AIPTS 2024 – PRELIMS TEST 1 (ANCIENT HISTORY) SOLUTIONS

As Buddhism travelled to new regions such as Sri Lanka, other texts such as the Dipavamsa
(literally, the chronicle of the island) and Mahavamsa (the great chronicle) were written,
containing regional histories of Buddhism. Many of these works contained biographies of the
Buddha. Some of the oldest texts are in Pali, while later compositions are in Sanskrit. (Hence
option (c) is correct answer)

Q.33) Solution (b)

Explanation:

Therigatha: A unique Buddhist text and part of the Sutta Pitaka, is a collection of verses
composed by bhikkhunis (female monks). It provides an insight into women’s social and spiritual
experiences during their times. (Hence option b) is correct answer)

Important Points:

Q.34) Solution (d)

Explanation:

Like the Charvakas, the Jains too do not believe in the Vedas, but they admit the existence of a
soul. They also agree with the orthodox tradition that suffering (pain) can be stopped by
controlling the mind and by seeking right knowledge and perception and by observing the right
conduct. They believed that the world is created and maintained by Universal Law. (Hence
option (d) is correct answer)
.
IASBABA’S AIPTS 2024 – PRELIMS TEST 1 (ANCIENT HISTORY) SOLUTIONS

Q.35) Solution (a)

Explanation:

● The early Jainas discarded Sanskrit and adopted Prakrit but later in early medieval times,
Jainas also made substantial use of Sanskrit and wrote many texts in it. (Hence statement
1 is incorrect)
● Many regional languages developed out of Prakrit, particularly Shauraseni from which the
Marathi language developed. (Hence statement 2 is correct)
● Initially, like the Buddhists, the Jainas were not image worshippers. (Hence statement 3
is incorrect)

Important points:

Jain Agamas

● They are sacred books that are supposed to contain the Jain tirthankaras' teachings.
● They are claimed to have been compiled by the Gandharas, Mahavira's direct disciples.
For Svetambaras, these scriptures are crucial.
● The present Angas are claimed to have been re-compiled in the mid-5th century AD in
Vallabhi (Gujarat) by a council of Svetambaras sect monks.
● The Digamabara sect thinks that the original teachings were lost a long time ago and does
not recognise the authority of the Agamas written in Valabhi.
● Agamas is a collection of 46 texts.
● There are twelve Angas, twelve Upangas, ten Prakirnakas, four Mulasutras, six
Chedasutras, and two Chulika Sutras.
● They were written in the Prakrit language Ardha-Magadhi.
● The Angas instill regard for all forms of life, rigorous vegetarianism, asceticism,
compassion, and nonviolence in their followers. There are 12 Angas in total.
● Non‑agam Literature:
● Commentary and explanations of Agam literature, as well as independent works gathered
by ascetics and scholars, make up non-agam literature.
● They are written in Prakrit, Sanskrit, Apabhramsa, Old Marathi, Rajasthani, Gujarati, Hindi,
Kannad, Tamil, German, and English, among other languages.

Q.36) Solution (b)

Explanation:
.
IASBABA’S AIPTS 2024 – PRELIMS TEST 1 (ANCIENT HISTORY) SOLUTIONS

● According to a late tradition, the spread of Jainism in Karnataka is attributed to


Chandragupta Maurya (322– 298 BC). The emperor became a Jaina, gave up his throne,
and spent the last years of his life in Karnataka as a Jaina ascetic, but this tradition is not
corroborated by any other source. The second cause of the spread of Jainism in south
India is said to have been the great famine that took place in Magadha 200 years after
Mahavira’s death. The famine lasted for twelve years, and in order to protect
themselves, many Jainas migrated to the south under the leadership of Bhadrabahu,
though the rest of them stayed back in Magadha under the leadership of Sthalabahu. This
emigrant Jainas spread Jainism in south India. (Hence statement 1 is correct)
● In order to spread the teachings of Jainism, Mahavira organized an order of his followers
that admitted both men and women. He preached his teachings in Prakrit, the language
of the common people. As Jainism did not very clearly differentiate itself from the
brahmanical religion, it failed to attract the masses. (Hence statement 2 is correct)

Q.37) Solution (c)

Explanation: All the statements are correct.

• Under the patronage of the Chola rulers, hundreds of temples were built in South India.
The Dravida style of architecture reached its climax under the Chola rulers.
• The Dravidian temples were surrounded by high boundary walls, the front wall had a high
entrance gateway known as the gopuram, a monumental entrance tower.
• One of the finest and most elaborate examples of the style is, however, provided by the
Brihadeswara temple at Tanjore built by Rajaraja I.

Q.38) Solution (d)

Explanation:

● Jainism was patronized by many dynasties, including Chandragupta Maurya, Kharavela of


Kalinga and the royal dynasties of south India, such as the Gangas, the Kadambas, the
Chalukyas and the Rashtrakutas. However, it was not patronized by the Cholas. The
Cholas patronized Hinduism, however, Jainism also flourished during their rule. (Hence
statement 1 is incorrect)
● The great famine that occurred in Magadha after the 200 years death of Mahavira also
contributed to its spread in southern India. According to tradition, the famine lasted for
about 12 years; and in order to protect themselves, many jaina followers under the
.
IASBABA’S AIPTS 2024 – PRELIMS TEST 1 (ANCIENT HISTORY) SOLUTIONS

leadership of Bhadrabahu shifted to Shravanabelgola in Karnataka. (Hence statement 2


is correct)

Q.39) Solution (d)

Explanation:

Reasons for decline of Jainism in India:

● For starters, the early pace of royal backing of Jainism by Bimbisara, Ajatasatru, Udayin,
and Kharavela was not maintained by successive kings and princes. Rather, Asoka,
Kanishka, and Harsha's fervour and resolve to propagate Buddhism began to overwhelm
Jainism.
● Split amongst Jainism into two sects i.e., the Digambaras (Sky-clad or naked) and
Swetambaras (clad in white)
● Lack of missionary zeal There was also a deterioration in the Jaina mendicants' missionary
enthusiasm and earnestness. They were no more picky in their efforts to promote Jainism
in villages and towns. The shopkeepers and businesspeople stayed committed to Jainism.
But they didn't have time to work on spreading Jainism.
● The revival of Brahmanism.
● Rigid principles and difficulty in following the severe penance and austerity.
● Spread of Buddhism as a rival faith led to the decline of Jainism

(Hence all the statements are correct and option (d) is the correct answer)

Q.40) Solution (c)

Explanation:

● Mahavira had eleven disciples, known as the Ganadharas or the heads of the schools.
Arya Sudharma was the only Ganadhara who survived Mahavira and became the first
'Thera' (The chief preceptor) of the Jaina order. (Hence statement 1 is correct)
● The Purvas contained the original doctrine in 14 text which Mahavira himself thought in
Ardhamagadhi to the disiciples. (Hence statement 2 is correct)
● Jainism has been divided into two major sects, Digambara and Swetambara. These sects
have been further sub divided into major and minor sub-sets.

The Digambara sect:


.
IASBABA’S AIPTS 2024 – PRELIMS TEST 1 (ANCIENT HISTORY) SOLUTIONS

● Major Sub-Sects
o Bisapantha
o Terapantha
o Taranapantha or Samaiyapantha (Hence statement 3 is correct)
● Minor Sub-Sects:
o Gumanapantha
o Totapantha

The Swetambara sect:

o Murtipujaka
o Sthanakvasi
o Terapanthi

Q.41) Solution (d)

Explanation:

● Buddhist monks in later years gave up Pali, the language of the people, and took to
Sanskrit, the language of intellectuals. (Hence statement 1 is correct)
● By the seventh century, the Buddhist monasteries had come to be dominated by ease-
loving people and became centres of corrupt practices which had been prohibited by
Gautama Buddha. (Hence statement 2 is correct)
● The Brahmin ruler Pashyamitra Shunga is said to have persecuted the Buddhists. Several
instances of persecution occur in the sixth–seventh centuries. The Huna king Mihirakula,
who was a worshipper of Shiva, killed hundreds of Buddhists. The Shaivite Shashanka of
Gauda felled the Bodhi tree at Bodh-Gaya where the Buddha had attained enlightenment.
Such incidents led to the fleeing of Buddhists to nearby regions like Nepal Bhutan and
China. (Hence statement 3 is correct)

Q.42) Solution (d)

Explanation:

There were various causes for the rise of Jainism and Buddhism:

● Religious factors - complex and expensive Vedic rituals, Upanishads being highly
philosophical in nature etc. (Hence statement 1 is correct)
.
IASBABA’S AIPTS 2024 – PRELIMS TEST 1 (ANCIENT HISTORY) SOLUTIONS

● Social factors - rigidity of caste system, domination of priestly class etc. (Hence statement
4 is correct)
● Economic factors - rise of trade and therefore, growing economic domination of Vaishyas,
sacrifice of animals leading to depletion of cattle wealth etc.
● Jainism and Buddhism were taught in Prakrit and Pali respectively thus were more
accessible to the common man as compared to Sanskrit. (Hence statement 2 is correct)

Q.43) Solution (b)

Explanation:

● Samkhya: According to the early Samkhya philosophy, the presence of divine agency is
not essential to the creation. The world owes its creation and evolution to the nature than
God. (Hence pair 1 is correctly matched)
● Mimamsa: This Ideology believed on reasoning and interpretation. The reasoning was
provided to justify the Vedic rituals. They believed that Vedas contain eternal truth. This
philosophy believed in Vedic sacrifices. (Hence pair 2 is incorrectly matched)
● Vaisheshika: This philosophy gave importance to the material elements (Earth, water,
fire, air, ether). These 5 elements combine to give rise to new elements. This school of
thought propounded the atom theory. (Hence pair 3 is correctly matched)

Q.44) Solution (a)

Explanation:

● The Indo-Greeks were also the first to issue gold coins in India, and these increased in
number under the Kushans. At the same time Indo-Greeks were the first rulers in India to
issue coins that can be definitively attributed to particular kings. (Hence statement 1 is
correct)
● Kanishka, who was the most famous ruler of the Kushan period, extended his
wholehearted patronage to Buddhism. He held a Buddhist council in Kashmir, where the
doctrines of the Mahayana form of Buddhism were finalized. The fourth Buddhist Council
was held under the patronage of Kanishka in Kashmir in 72 AD. It was conducted to
systematize the Sarvastivadin Abhidharma texts, which were translated from earlier
Prakrit vernacular languages into the classical language of Sanskrit. (Hence statement 2
is incorrect)
.
IASBABA’S AIPTS 2024 – PRELIMS TEST 1 (ANCIENT HISTORY) SOLUTIONS

● The Government of India uses the Shaka era as an official calendar but it was not started
by Shaka rulers. Instead, the Shaka era was started by Kanishka, who was a Kushan ruler.
(Hence statement 3 is incorrect)

Important Points:

• The national calendar based on the Saka Era, with Chaitra as its first month and a normal
year of 365 days was adopted from 22 March 1957 along with the Gregorian calendar for
the following official purposes:
• Gazette of India.
• News broadcast by All India Radio.
• Calendars issued by the Government of India.
• Government communications addressed to the public.
• Dates of the national calendar have a permanent correspondence with dates of the
Gregorian calendar, 1 Chaitra falling on 22 March normally and on 21 March in leap year.

Q.45) Solution (a)

Explanation:

● The play Mudrarakshasa by Visakhadatta was written during the Gupta period, sometime
after the 4th century CE.
● It narrates Chandragupta’s accession to the throne of the Magadha Empire and the
exploits of his chief advisor Chanakya or Kautilya by listing the strategies he used to
counter an invasion against Chandragupta. (Hence option (a) is correct answer)
● This play is often cited as a corroborative source since it supports the information
gathered from other contemporary sources about Chandragupta.

Important points:

• Megasthenes' Indica is a history of Mauryan India written by the Greek writer


Megasthenes.
• The original work has since been lost, although pieces of it have survived in later Greek
and Latin works.
• The Arthashastra is a Sanskrit book on statecraft, political science, economic policy, and
military strategy written in ancient India. The text's author is typically assigned to Kautilya,
also known as Vishnugupta and Chanakya. Emperor Chandragupta Maurya's instructor
and guardian was the latter, a scholar at Takshashila. It's possible that Kautilya's
.
IASBABA’S AIPTS 2024 – PRELIMS TEST 1 (ANCIENT HISTORY) SOLUTIONS

Arthashastra is the most methodical, scholarly, and authoritative account of these archaic
investigations.
• Dipavamsa: The Dīpavaṃsa is the oldest historical record of Sri Lanka. The chronicle is
believed to be compiled from Atthakatha and other sources around the 3rd to 4th century
CE. Together with the Mahāvaṃsa, it is the source of many accounts of the ancient history
of Sri Lanka and India.

Q.46) Solution (d)

Explanation:

● The richer landowners were called grihapatis. These landowners employed labourers
called dasas or karmakaras. The smaller landowners were known as kassakas or
krishakas.
● The society was stratified on the basis of varna. It emerged as a marker of status.
Cultivators and artisans were identified as the shudras. A new social category that
emerged during this period was placed below the shudras in the social hierarchy and
considered untouchables. They were forced to live on the fringes of the settlements and
subsisted on hunting and gathering their food. They were marginalised and given only
menial jobs as urbanisation was on the rise.

Q.47) Solution (c)

Explanation:

● The Buddhist text, Anguttara Nikaya, which is a part of the Sutta-Pitaka, gives the list of
16 Mahajanapadas. (Hence statement 1 is correct)
● The Mahajanapadas represented a conglomerate of thousands of villages and a few cities.
These Mahajanapadas extended from the North-Western Pakistan to East Bihar and from
the sub-montane regions of the Himalayas to river Godavari in the South. (Hence
statement 2 is incorrect)
.
IASBABA’S AIPTS 2024 – PRELIMS TEST 1 (ANCIENT HISTORY) SOLUTIONS

Q.48) Solution (d)

Explanation:

Nalanda became a great centre of learning during Harsha’s reign. According to HiuenTsang,
Nalanda housed as many as 10,000students. It was supported by the revenues of 200 villages.
Although this huge monastic educational establishment was primarily a centre for learning of
Mahayana Buddhism, yet the curriculum included secular subjects as well. Grammar, logic,
.
IASBABA’S AIPTS 2024 – PRELIMS TEST 1 (ANCIENT HISTORY) SOLUTIONS

epistemology and sciences were taught here. Harsha is said to have invited a thousand learned
monks of Nalanda to take part in the philosophical assembly at Kanauj. In his account, Hieun-
Tsang has given a detailed account of Nalanda. (Hence option d) is correct answer)

Q.49) Solution (b)

Explanation:

Mahajanapadas were of two types:

● Monarchial states- Anga, Avanti, Chedi, Kashi, Kosala, Gandhara, Magadha, Matsya,
Surasena, Vatsa
● Non-Monarchial/Republican states- Kamboj, Kuru, Koliyas, Malla, Moriya, Shakya, Vajji,
Panchal, Licchavi, Bhaggas, Kalamas, Videhas, Jnatrikas.
● From the above, it is clear that Magadha is not a Republic states.

Q.50) Solution (c)

Explanation:

● The Mauryas created for the first-time well-organized state machinery, which operated
in the heart of the empire. It constitutes a landmark in the system of taxation in ancient
India. Kautilya, names many taxes to be collected from peasants, artisans and traders. The
Mauryas attached greater importance to assessment than to storage and depositing.
Therefore, an officer designated as ‘Samaharta’ was appointed as the highest officer in
charge of assessment and the ‘Sannidhata’ was the chief custodian of the state treasury
and store-house. (Hence statement 1 is correct)
● The punch marked silver coins, which carry the symbols of the peacock, and the hill and
crescent, also formed the imperial currency of the Mauryas. These have been discovered
in large numbers. (Hence statement 2 is incorrect)

Tips: You can Eliminate the options with extreme statements.

Q.51) Solution (c)

Explanation:
.
IASBABA’S AIPTS 2024 – PRELIMS TEST 1 (ANCIENT HISTORY) SOLUTIONS

● Contemporary accounts by Greek historians show that Chandragupta was a youth living
in Taxila when Alexander invaded India. Greek historians have recorded his name as
“Sandrakottus” or “Sandrakoptus”, which are evidently modified forms of Chandragupta.
● Inspired by Alexander, Chandragupta led a revolt against the Nandas years later and
overthrew them.

Important Points:

• Justin, the Greek writer of “Epitome” had mentioned Chandragupta Maurya as


Sandrocottus. It was identified by William Jones. Justin mentioned that Chandragupta
Maurya had visited Alexander's court. Ashoka is mentioned as Piyadasi in many of his
inscriptions.

Q.52) Solution (d)

Explanation:

● Craft products like textile goods, ivory products, pots, etc., were exported over long
distances by merchants. All the major cities were situated on river banks and trade routes,
and were connected with each other.
● The two major trans-regional routes of the time were known as Uttarapatha (of northern
India, stretching from the north-west across the Indo-Gangetic plains to the port city of
Tamralipti on the Bay of Bengal). (Hence statement 1 is incorrect)
● Dakshinapatha (of southern India, stretching from Pataliputra in Magadha to
Pratishthana on the Godavari, and connected to ports on the western coast). (Hence
statement 2 is incorrect)

Important Points:

• The Uttarapatha has also been mentioned by Acharya Panini and in Ashtadhyayi. Many
Jataka stories of the Buddhists also mentions this route.
• Dakshinapath has been mentioned in Arthashastra. It was connected with the
Uttarapatha in Kaushambi.
• Many Mahajanapadas around the Ganga valley were located around and about this very
Uttarapatha.
• Gandhara benefitted too much from this route as it was located straight on the route.
Alos Mahajanapadas like Kuru, Matsya, Kosala, Magadha etc were situated on the
Northern part of the Silk Route of India. Only Avanti was situated on the southern route.
.
IASBABA’S AIPTS 2024 – PRELIMS TEST 1 (ANCIENT HISTORY) SOLUTIONS

Q.53) Solution (a)

Explanation:

● Disregarding the popular view that our planet earth is Achala (immovable), Aryabhatta
stated his theory that the earth is round and rotates on its own axis. He explained that
the appearance of the sun moving from east to west is false. (Hence statement 1 is
incorrect)
● Baudhayana was the first one ever to arrive at several concepts in Mathematics, which
were later rediscovered by the western world. He made reference of Pi before any other
scholars. Later, Aryabhata discovered an approximation of pi, 62832/20000 =
3.1416.(Baudhyana was not first to calculate the value but first to discover pi) What is
known as Pythagoras theorem today is already found in Baudhayan’s Sulva Sutra. (Hence
statement 2 is correct and 3 is incorrect)

Q.54) Solution (a)

Explanation:

● The administration of Pataliputra, which was the capital of the Mauryas, was carried out
by six committees. These committees were entrusted with sanitation, care of foreigners,
registration of birth and death, regulation of weights and measures and similar other
functions. Various types of weights belonging to Mauryan times have been found at
several places in Bihar. (Hence statement 1 is correct)
● According to the account of a Roman writer called Pliny, Chandragupta maintained
600,000 foot-soldiers, 30,000 cavalry and 9000 elephants. In addition to this the
Mauryans maintained a navy and chariots also. (Hence statement 2 is correct)
● In Arthashastra of Kautilya it would appear that the state controlled almost all the
economic activities in the realm. It seems that taxes collected from the peasants varied
from one-fourth to one-sixth of the produce. Those who were provided with irrigation
facilities by the state had to pay for it. (Hence statement 3 is incorrect)

Q.55) Solution (c)

Explanation:

● In 305 B.C., Chandragupta Maurya marched against Selukas Niketar, who was
Alexander’s General controlling the north western India. Chandragupta Maurya defeated
.
IASBABA’S AIPTS 2024 – PRELIMS TEST 1 (ANCIENT HISTORY) SOLUTIONS

him and a treaty was signed. By this treaty, Selukas Niketar ceded the trans-Indus
territories – namely Aria, Arakosia and Gedrosia – to the Mauryan Empire. He also gave
his daughter in marriage to the Mauryan prince. Chandragupta made a gift of 500
elephants to Selukas. (Hence statement 1 is incorrect)
● Greek writers address Bindusara as Amitrochates (destroyer of foes), while the
Mahabhasya refers to him as Amitraghat (killer of enemies). This name was given to him
because like his father Chandragupta Maurya he was also a powerful king and merciless
for his enemies. (Hence statement 2 is correct)
● Chanakya assisted the first Mauryan emperor Chandragupta Maurya in his rise to power
and is widely credited for having played an important role in the establishment of the
Maurya Empire. Chanakya served as the chief advisor in the administration to both
Emperors Chandragupta Maurya and his son Bindusara. (Hence statement 3 is correct)

Q.56) Solution (a)

Explanation:

● The lower varnas were allowed to hear religious books. They could worship of gods like
Krishna. They could conduct domestic religious rights. During earlier periods they were
not allowed to perform these activities. (Hence statement 1 is correct)
● The woman of upper varnas were not allowed to work independently unlike the lower
varnas woman. The vaishyas and shudra women took agricultural works. (Hence
statement 2 is incorrect)

● The practise of untouchability had begun during the Gupta period. The Chinese traveller
Fahien mentions that 'Chandalas' were separated from society. So Chandals were
untouchables outside the four Varna system. (Hence statement 3 is incorrect)

Q.57) Solution (d)

Explanation:

● Megasthenes states that he did not notice any slaves in India. (Hence statement 1 is
incorrect)
● The royal road ran from Pataliputra to Nepal through Vaishali and Champaran. Also, roads
are found at the foothills of the Himalayas which passed from Vaishali through
Champaran to Kapilavastu, Kalsi (in Dehra Dun district), Hazra, and eventually to
.
IASBABA’S AIPTS 2024 – PRELIMS TEST 1 (ANCIENT HISTORY) SOLUTIONS

Peshawar. Megasthenes speaks of a road connecting northwestern India with Patna as


well. (Hence statement 2 is correct)

Q.58) Solution (c)

Explanation:

● The administrative mechanism was backed by an elaborate system of spy. Various types
of spies collected intelligence about foreign enemies and kept an eye on numerous
officers. (Hence statement 1 is correct)
● King was the supreme source of all powers and was center of all authorities, judiciary and
administration. In the highly centralized administration, King used to select ministers,
high officials and established a well-planned system of supervision and inspection. (Hence
statement 2 is incorrect)

Q.59) Solution (b)

Explanation:
.
IASBABA’S AIPTS 2024 – PRELIMS TEST 1 (ANCIENT HISTORY) SOLUTIONS

● During Mauryan Emperor Ashoka, the political unity was achieved for the first time in the
Indian history under the Mauryas reign, with the conquest of the Indo-Gangetic plain, the
empire was centralized. (Hence statement 1 is correct)
● The bali which means land tax was the main item of revenue and was levied at the rate
of one-sixth of the produce and not more than that. The land revenue was fixed at one-
sixth (shadabhaga) of the product and was collected by the revenue officer (Agronomoi).
The toll tax was roughly around ten percent. (Hence statement 2 is incorrect)
● There was a provision for Census on regular basis in the Mauryan age. It appears to be
the permanent institution in the Mauryan administration. There were village officials who
record the number, caste, occupation, and other details of people. The count of animals
is also in each house. The municipal officers were appointed for the census in town. They
were to track the movement of the population both foreign and indigenous. (Hence
statement 3 is correct)

Q.60) Solution (d)

Explanation:

● There was an adverse Brahmanical reaction against the policies of Ashoka. Although
Ashoka adopted a tolerant policy and asked the people to respect even the brahmanas,
but he issued his edicts in Prakrit and not in Sanskrit, which generated a reaction from
Brahmanas. (Hence statement 1 is correct)
● Ashoka made large donations to the Buddhist monks which left the royal treasury empty.
(Hence statement 2 is correct)
● Since Ashoka was primarily pre-occupied with missionary activities at home and abroad,
he was unable to pay attention to safeguarding the passes through the north-western
frontier.

● The Chinese ruler Shih Huang Ti (247-210 BCE) constructed the Great Wall of China in
about 220 BCE, to protect his empire against the attacks of the Scythians, a central Asian
nomadic tribe who were in a state of constant flux.

● No such measures were taken by the emperor Ashoka on the northwestern frontier of
India.

● In order to escape the Scythians, the Parthians, the Shakas and the Greeks were forced to
move towards India.
.
IASBABA’S AIPTS 2024 – PRELIMS TEST 1 (ANCIENT HISTORY) SOLUTIONS

● The Greeks were the first to invade India in 206 BCE and they set up their kingdom in
north Afghanistan called Bactria.

● This was followed by a series of invasions till the beginning of the Christian era. (Hence
statement 3 is correct)

Q.61) Solution (d)

Explanation:

Literature of Gupta's Age

• The Gupta age constitutes a new epoch in the history of Sanskrit literature. It reached its
perfection in both form and content, and almost all the best works in its different
branches were composed before the end of this glorious period. (Hence statement 1 is
correct).

• It may be called the great age of Sanskrit also from another point of view. For Sanskrit
now replaced Prakrits not only in epigraphic, but even in the religious and philosophical
literature of the heterodox sects like the Jainas and Buddhists. (Hence statement 2 is
correct).

• The continuity of the literary tradition is best marked in the field of Brahmanical literature.
For one of the foremost literary activities of the Gupta period is to be seen in the final
redaction of at least one of the two great Epics, viz. the Mahabharata, as well as the
development of the Puranas and Smriti or Dharmashastra literature. (Hence statement 3
is correct).

Important Points:

• Kiratarjuniya, which talks about the conversation of Shiva and Kiratarjuniya, an epic style
Kavya is considered to be among the greatest works in Sanskrit was written during this
period by Bharavi.

Q.62) Solution (a)

Explanation:

Samudra Gupta
.
IASBABA’S AIPTS 2024 – PRELIMS TEST 1 (ANCIENT HISTORY) SOLUTIONS

• His competent leadership resulted in some high-quality gold coins, laying the groundwork
for India's Golden Age.

• During his reign, he is said to have issued only gold coins (Dinar) in seven different types
(including the 'Lichhaviya' type).

• The coins of Samudragupta reveal a great deal about the beginnings of the Gupta dynasty
and its economy.

• In numismatic terms, Samudragupta's coins are classified as follows, based on their design
and variety:

• Standard type: The standard types are many and widely used. The king is depicted
holding a Garuda Dhwaja in his left hand and wearing a hat in this sort of currency.
The goddess Laxmi is shown on the coin's reverse side.

• The Archer type: The Archer types, which are rare and depict Samudra Gupta
holding the bow, were introduced for the first time in Indian Numismatics.

• Battle axe type: Weapons such as the battle axe, bow, arrow, and swords were
featured on Samudragupta's coins.The legend "Kritantparashuh" appears on his
battle axe type.

• Lichchavi type: The Licchavi highlights the main King Chandragupta-I and his
Lichchavi family queen Kumaradevi. Despite the fact that the legend is specific to
Chandragupta I, Samudragupta issued it in commemoration of his father.

• Kacha type: The inscription on the front of the Kacha-type coin reads, "Kacha,
having conquered the earth, gains heaven by the highest works," while the reverse
reads, "Exterminator of all the monarchs."

• The tiger slayer type: The king's tiger slayer coins depict him crushing a tiger while
firing a bow at it. "Vyagraparakramah," says the obverse legend.

• The lyrist type: In the Lyrist type, the king is seated on a high-backed couch,
playing the Veena on his knees. The front bears the legend "Maharajadhiraja – Sri
Samudragupta."

• Asvamedha type: Asvamedha types are one-of-a-kind; we see a horse standing in


front of a yupa or a sacrificial post, surrounded by legends proclaiming the King to
be the conqueror of heaven, earth, and the oceans.
.
IASBABA’S AIPTS 2024 – PRELIMS TEST 1 (ANCIENT HISTORY) SOLUTIONS

• In comparison to the older Kushan currency, Samudragupta's coinage had a uniquely


Indian flavor to it in terms of the representation of clothes, weapons, gods, and so on.

• Shooting at a lion is not included in the types of coins which was issued by Samudragupta.
(Hence type 4 is incorrect)

Q.63) Solution (a)

Explanation:

• Vasudeva I: He was the last great Kushan emperor, and the end of his rule coincides with
the invasion of the Sasanians as far as northwestern India, and the establishment of the
Indo-Sasanians or Kushans in what is nowadays Afghanistan, Pakistan and northwestern
India from around AD 240. (Hence pair 1 is incorrectly matched)

• The last of the Shunga emperors was Devabhuti (83–73 BCE). He was assassinated by his
minister (Vasudeva Kanva) and is said to have been over fond of the company of women.
The Shunga dynasty was then replaced by the subsequent Kanvas. The Kanva dynasty
succeeded the Shungas around 73 BCE. (Hence pair 2 is incorrectly matched)

• Pulumavi IV, the last king of the main line, ruled until c. 225 CE. During his reign, several
Buddhist monuments were constructed at sites including Nagarjunakonda and Amaravati.
Madhya Pradesh was also part of his kingdom. (Hence pair 3 is correctly matched)

Q.64) Solution (c)

Explanation:

● Mayurasharman, the first Kadamba king, was crowned by the Pallavas. This happened in
AD 350. He is known as Mayuravarman to medieval records who performed eighteen
ashvamedhas.
● Senaguttuvan was succeeded by his half- brother Perunjeral (or Perumcheral) Adan (c. AD
180) known to Patirrupattu as Adu-Kotpattu Cheralatan. He was the contemporary of the
great Chola king Karikal.
● The Battle of Vatapi was a decisive engagement which took place between the Pallavas
and Chalukyas near the Chalukya capital of Vatapi (present day Badami) in 642. The battle
resulted in the defeat of the Chalukya king Pulakeshin II in 642 CE and the commencement
of Pallava occupation of Vatapi lasted until 654.
.
IASBABA’S AIPTS 2024 – PRELIMS TEST 1 (ANCIENT HISTORY) SOLUTIONS

● Gopala (750-770 AD) was the founder of the Pala Dynasty in Eastern Part of the India. He
laid the foundation of Odantpuri Mahavihara.

Q.65) Solution (d)

Explanation:

● It was Srigupta who has been identified with the first Gupta king of that name in Allahabad
Inscription. The Allahabad Inscription also mentions of maharaja Srigupta and maharaja
Ghatotkacha as the ancestors of Samudragupta. (Hence Statement 1 is incorrect)
● After the end of the Satavahanas and Kushanas empires in the middle of third century
A.D., the empire of Gupta emerged. Thus, it was not the contemporary of Satavahanas
and Kushanas. (Hence Statement 2 is incorrect)
● It was mentioned in Puranas and not in the Allahabad Inscription. It mentions the four
regions i.e. Magadha, Saketa (Ayodha), Prayag and Gangetic Basin. (Hence Statement 3
is incorrect)

Important Points:

• Vaktaka dynasty was the most important follow up dynasty of the Satavahanas and they
ruled in modern Maharastra and Madhya Pradesh. They were contemporary of Guptas.
Vindhyashakti, whose name is derived from the Vidhya Mountains, founded Vaktaka
dynasty. Pravarasena was an able ruler who is also known to be the real founder of this
dynasty.
• One of the important rulers of this dynasty was Rudrasena I who is mentioned in the
Allahabad Pillar Inscription. Rudrasena I was followed by Prathivisena I. During the
campaign of Chandragupta II against the western Satraps, Chandragupta II is considered
to have alliance of Prathivisena I and later cemented this tie with a marital alliance giving
his daughter Prabhavatigupta to Rudrasena II, the prince of Vaktaka Family. The capital of
Vaktataka during the reign of Prathivisena and Rudrasena II was Nandivardhana, which is
near modern Nagpur.

Q.66) Solution (a)

Explanation:
.
IASBABA’S AIPTS 2024 – PRELIMS TEST 1 (ANCIENT HISTORY) SOLUTIONS

● Akshapataladhyaksha was the Accountant General who was in charge of the two offices
of Currency and Accounts. The fiscal year was from Asadha (July), and 354 working days
were reckoned in each year. (Hence pair 1 is correctly matched)
● Sitadhyaksha was the Director of Agriculture and in charge of the cultivation of Crown
lands or of Government agricultural farms. (Hence pair 2 is correctly matched)
● Akaradhyaksha was the Superintendent of mining and possessed scientific knowledge
of mines, metallurgy, gems, and precious stones. The State had a monopoly in the working
of mines and in in their products.
● Lavanadhyaksha was the Salt Superintendent. The manufacture of salt was a
Government monopoly administered by the Salt Superintendent. It was administered
through a system of licences, on payment of a fixed fee, or a share of the output. (Hence
pair 3 is incorrectly matched)
● Navadhyaksha was the Superintendent of Ports. He controlled traffic and transit by
waterways, riverine, oceanic, coastal routes, etc. and also policed the rivers and
seashores.

Other Important Officials:


.
IASBABA’S AIPTS 2024 – PRELIMS TEST 1 (ANCIENT HISTORY) SOLUTIONS

Q.67) Solution (d)

Explanation:

● Brahmanical literature was particularly severe in the treatment of women and assigned
to them a very low status in the society. Buddhist texts, on the other hand, were much
more considerate in treating them.
.
IASBABA’S AIPTS 2024 – PRELIMS TEST 1 (ANCIENT HISTORY) SOLUTIONS

● Megasthenes testifies to the growing practice of polygamy; employment of women as


palace guards, bodyguards to the king, spies, etc; permission of widow remarriage and
divorce. (Hence all the statements are correct)
● Thus, the position of women, though inferior to that of men and witnessing a downtrend,
was not as bad as it came to be in the later periods such as the Gupta period.

Q.68) Solution (a)

Explanation:

● Shunga dynasty, Indian ruling house founded by Pushyamitra about 185 bce, which
replaced the Mauryan dynasty. Pushyamitra assassinated Brihadratha, the last Mauryan
ruler, at a military parade and assumed royal power. (Hence statement 1 is correct)
● Pushyamitra was succeeded by his son Agnimitra. Agnimitra is the hero of Kalidasa's
Malavikagnimitra. Agnimitra was succeeded by Sujyeshtha, identical with Jethamitra of
the coins. After him came Agnimitra's son, Vasumitra, who was soon murdered by
Muladeva of Kosala. (Hence statement 3 is incorrect)
● The Sunga period witnessed the revival of Brahmanism and the growing importance of
the Bhagavata religion. The great grammarian, Patanjali, born at Gonarda in central
India, was a contemporary of Pushyamitra. (Hence statement 2 is correct)

Q.69) Solution (b)

Explanation:

● The Indo-Greek kingdom was ruled by over 30 Hellenistic (Greek) kings in the northwest
and north India from the 2nd century BC to the beginning of the first century AD. The
kingdom started when Greeco-Bactrian king Demetrius (son of Euthydemus I) invaded
India around 180 BC.
● The first Saka king of India was Maues/Moga (1st century BCE) who established Saka
power in Gandhara, Afghanistan, Pakistan, India and the Indus Valley. The Indo-Scythians
extended their supremacy over north-western India, conquering the Indo-Greeks and
other local kingdoms.
● The Indo-Parthian Kingdom was a Parthian kingdom founded by Gondophares, and active
from 19 CE to 226 CE. At their zenith, they ruled an area covering parts of eastern Iran,
various parts of Afghanistan and the northwest regions of the Indian subcontinent (most
of modern Pakistan and parts of northwestern India).
.
IASBABA’S AIPTS 2024 – PRELIMS TEST 1 (ANCIENT HISTORY) SOLUTIONS

● In the 1st century CE, Kujula Kadphises (Kadphises I) brought together these five
principalities and founded the Kushan Empire. The Kushans movement in India can be
traced back to the first century CE during Kadphises I time.
● Hence correct sequence is Indo-Greeks, Indo-Scythians Indo-Parthians, Kushans

Q.70) Solution (b)

Explanation:

Differences between Indus Valley and Mesopotamian civilizations

● In Harappan civilization, the houses were built to the East of citadel while in
Mesopotamian civilization, the houses were built around distinct palaces and temples &
dedicated to the patron Gods or Goddesses.
● While theory of Kingship or a ruler class has not been proved for Indus Valley, the
Mesopotamians were ruled by a priest King, who was considered to be divine. That King
solved disputes led administration and had an army too. (Hence statement 2 is correct)
● While no special places of worship have been found in Indus Valley, it is assumed that
these people worshipped nature. However, presence of different religious objects such
as seals, statues etc. indicate that Indus Valley might be having a polytheistic religion. In
Mesopotamia also, polytheism was present but the Mesopotamians also built temple for
worship. (Hence statement 1 is correct)
● The Harappan people were peaceful and lacked a strong military presence. This may have
been due to the fact that they relied more on trade than warfare for survival. Whereas
Mesopotamians were much better equipped for combat, their armies consisted of
chariots driven by horses, which made them quite capable of attacking enemy
strongholds or routing opposing forces.

Q.71) Solution (b)

Explanation:

● Jainism recognised the existence of God while Buddhism did not. (Hence statement 1 is
correct)
● Jainism does not condemn the Varna system while Buddhism does. (Hence statement 2
is incorrect)
● Jainism believed in transmigration of soul i.e. reincarnation while Buddhism does not.
(Hence statement 3 is correct)
.
IASBABA’S AIPTS 2024 – PRELIMS TEST 1 (ANCIENT HISTORY) SOLUTIONS

● Buddha prescribed the middle path while Jainism advocates his followers to even
completely discard the clothes i.e. life of austerity.

Q.72) Solution (b)

Explanation:

The edicts gave Asoka the opportunity to expound his dhamma. While different major rock edicts
talk about different aspects of the dhamma, the Major Rock Edict XI contains an elaborate
explanation of the dhamma, apart from dealing with charity and kinship of humanity. It clearly
indicates that dhamma was a secular. (Hence option b is correct)

• Major Rock Edict V- About Dhammamahamatras. Talks about treating slaves right. A
special cadre of officials, Dhamma Gosha were appointed and entrusted with the duty of
spreading Dhamma within the kingdom.
• Major Rock Edict XIII- Mentions victory over Kalinga. Mentions Ashoka’s Dhamma victory
over Greek Kings Antiochus of Syria (Amtiyoko), Ptolemy of Egypt (Turamaye), Magas of
Cyrene (Maka), Antigonus of Macedon (Amtikini), Alexander of Epirus (Alikasudaro). Also
mentions Pandyas, Cholas, etc.
• The thirteenth rock edict which was issued at the end of the Kalinga war gives a vivid
picture of the change of Ashoka from an aggressive and violent warrior to a great lover
and preacher of peace. The direct and immediate effect of the Kalinga war was the
conversion of Ashoka to Buddhism.
• Major Rock Edict X- Disapproves of the individual’s desire for fame and glory and stresses
on Dhamma.

Q.73) Solution (b)

Explanation:

• Lumbini Pillar Edict in Nepal is known as the Rummindei Pillar Inscription. The Lumbini
Pillar Edict recorded that sometime after the twentieth year of his reign, Ashoka travelled
to the Buddha's birthplace and personally made offerings. He then had a stone pillar set
up and reduced the taxes of the people in that area. The pillar was erected by Asoka to
commemorate the birthplace of the Buddha, the Lumbini grove. (Hence option b is
correct).
.
IASBABA’S AIPTS 2024 – PRELIMS TEST 1 (ANCIENT HISTORY) SOLUTIONS

Q.74) Solution (b)

Explanation:

Religious aspect of Satavahana Dynasty:

● The Satavahanas claimed to be Brahmanas.


● The kings and queens of the Satavahana Dynasty performed Ashvamedha Vajapeya
(Horse sacrifice).
● Vaishnava Gods such as Krishna and Vasudeva were worshiped largely by the
Satavahanas. (Hence statement 3 is correct)
● The rulers gave the Buddhist monks tax-free lands and promoted Buddhism. The
Satavahanas are known for starting the practice of giving royal grants of land to Brahmans
and Buddhist monks. (Hence statement 1 is correct)
● Mahayana's form of Buddhism was followed widely in the Satavahana Empire. (Hence
statement 2 is incorrect)
● Under the reign of the Satavahana Dynasty, Nagarjunakonda and Amaravati became the
seat of Buddhist culture.

Q.75) Solution (d)

Explanation:

● Harshavardhana, the greatest king of Pushyabhuti dynasty. He was one of the most
important Indian emperors of the 7th Century. During the peak of his reign, his empire
extended from north India to the Narmada River in central India. (Hence statement 1 is
correct).
● During Harsha’s period, ministers and high ranking officers of the state were endowed
with land. The feudal practice of rewarding and paying officers with grants of land seems
to have begun under Harsha. (Hence statement 2 is correct)
● Caste system was prevalent among Hindus. They were divided into four castes or varna:
Brahmana, Vaishya, Kshariya and Shudra, which among them had their own sub-castes.
Hiuen-Tsang also mentions the existence of untouchables and outcastes. (Hence
statement 3 is correct)

Q.76) Solution (d)

Explanation:
.
IASBABA’S AIPTS 2024 – PRELIMS TEST 1 (ANCIENT HISTORY) SOLUTIONS

Religious beliefs of Harappan people:

● Worship of the Mother Goddess


● Worship of a male deity, most likely to be of Lord Shiva
● Worship of animals, semi human, or fabulous
● Worship of trees in their natural state or of their indwelling spirits
● Worship of inanimate stones or other objects, of linga and yoni symbol
● Believe in magic, charms and sacrifices

More about religion of Harappan Civilization:

● Along with male and female deities, the Indus people most likely worshipped Mother
Goddess.
● They worshiped a father God who was likely a founder of the race and a prototype of Siva
as the Lord of the Animals.
● They were familiar with some type of yoga and meditation.
● They believed in some form of tree of life, portrayed on seals as a Pipal or Acacia tree,
protected by a guardian spirit against an evil power signified by a tiger.
● The guardian spirit is shown in seals as a bull, a serpent, a goat, a legendary creature, or
an animal.
● They worshiped fertility symbols like round stones and perforated stones, which predated
the worship of Siva and Parvathi in the form Sivalinga.
● They might have believed in magical rites, charms, and amulets, as well as ghosts and
demons.
● They typically cremated the dead and left certain artefacts as offerings for their afterlives.
● The great bath of Mohenjo-Daro, or the much larger one discovered recently at the
Dholavira site in Kutch, was most likely a prototype temple ponds, or sacred tank, found
in ancient temples of southern India, where people might have taken purification baths
or collectively participated in some kind of ritual bath on important occasions.
● The presence of baths and the presence of animals in the Indus seals imply that they may
have utilized water and animals in sacrifice rituals as offerings or for expiation and
ceremonial cleaning.
● Seal in Mohenjo Daro with horned animal was thought to be one of the oldest portrayals
of the Hindu god Shiva, with "Pashupati" (Lord of Animals) being one of his epithets, or a
"proto-Shiva" deity.
● Though the mix of features in the Pashupati seal is unique, some of them can be found in
other Indus seals.
.
IASBABA’S AIPTS 2024 – PRELIMS TEST 1 (ANCIENT HISTORY) SOLUTIONS

Q.77) Solution (b)

Explanation:

Some of the Gupta’s inscriptions are characterized as prasasti and some are Royal charters.

Names of Inscriptions Character

● Allahabad Stone Pillar - Prasasti


● Eran Stone Pillar - Prasasti
● Nalanda Copper Plate - Royal Charter
● Gaya Copper Plate - Royal Charter

Q.78) Solution (a)

Explanation:

Classification of Land: From the economic standpoint, land of the Gupta period can be classified
into the following groups:

● Kshetra: Cultivatable land


● Khila: Waste land
● Aprahata: Jungle or forest land
● Vasti: Habitable land
● Gapata Sarah: Pasture land

Q.79) Solution (c)

Explanation:

Nalanda Vs Taxila

● Both Nalanda and Taxila universities were renowned ancient institutions of higher
learning. However, while Nalanda was a formal university in the modern sense of the
word, Taxila functioned under more informal conditions. (Hence statement 1 is correct)
● Nalanda had almost all the infrastructure we associate with a good university in today's
time, namely, multiple classrooms, extensive library, dormitories for students,
accommodation for professors, etc. On the other hand, Taxila University did not have any
lecture halls nor did it provide residential quarters for its teachers or students. (Hence
statement 2 is incorrect).
.
IASBABA’S AIPTS 2024 – PRELIMS TEST 1 (ANCIENT HISTORY) SOLUTIONS

● There was no centralized system of schooling or syllabus in Taxila. Taxila followed no


system of examinations, and did not award degrees to its students. The education system
there was flexible, and was modified according to a student's capabilities. Also, the
teachers of Taxila worked with complete autonomy, forming their own school of learning
with their specific set of rules and manner of functioning. Most of these schools were run
by teachers in their private houses, and received no formal infrastructural support from
the Taxila University, unlike Nalanda.
● As far as the objective of imparting knowledge is concerned, both Taxila and Nalanda
universities were very effective institutions, but their manner of functioning differed
drastically, with Nalanda University being more structured than Taxila.

● Nalanda university had disciplines of Astronomy, Mathematics, Politics and Science; while
Taxila university was mainly associated with teaching of Vedic literature and art skills like
archery and hunting only. (Hence statement 3 is not correct).

● Scholars like Nagarjuna, Panini, Chanakya, Prasanajit, and Jeevka (son of Bimbisara) all
came to Taxila to study before they became important personalities in Indian history.
Alexander took some scholars from Taxila to Greece. While Dinnaga (one of the Buddhist
founders of Indian logic), Dharmapala (second ruler of the Pala Empire of Bengal region)
were great scholars from Nalanda.

Q.80) Solution (c)

Explanation:

● The importance of irrigation to help agriculture was recognised in India from the earliest
times, for all parts of India were not liberally and equally watered by rivers and rainfall.
● The north- western part of India especially needed irrigation, which was done by various
means. Canals were constructed to prevent inundation as well as to irrigate dry areas.
● According to Narada, there were two kinds of dykes-the bardhya which protected the
field from floods and the khaya which served the purpose of irrigation. (Hence option c)
is correct)

Q.81) Solution (d)

Explanation:
.
IASBABA’S AIPTS 2024 – PRELIMS TEST 1 (ANCIENT HISTORY) SOLUTIONS

Prices and Measurements the Gupta period differed from the Mauryan age in one important
respect. Unlike the Mauryas, the Guptas did not try to fix the prices, which, therefore, fluctuated
according to supply and demand. In the Gupta period not only prices but also units of
measurements varied from place to place. (Hence statement 1 is incorrect and statement 2 is
correct).

Q.82) Solution (a)

Explanation:

Religion under Harsha

● Brahmanism, which reasserted itself under the Guptas, got further strengthened during
this period. (Hence statement 1 is correct).
● Its gradual ascendancy brought about the decline of Buddhism despite the patronage
given to it by Harsha which is evident from the account of Hiuen Tsang. But Jainism did
not undergo any major changes and it made neither progress nor any decay. (Hence
statement 2 is correct).
● Shaivism became the main theistic of this period. But Vaishnavism, which was very
popular during the age of the Guptas, was gradually declining during this period as is
evident from the rare references to it. (Hence statement 3 is incorrect).
● With a view to popularize and propagate the doctrines of Mahayana Buddhism, Harsha
arranged at Kanyakubja a great assembly which was presided over by Hiuen Tsang, in
643 CE. (Hence statement 4 is correct).
● The Vedic ceremonies and rituals once again came to be regarded as inseparable and
integral constituents of Brahmanism, and the people practised them on a large scale.

Q.83) Solution (c)

Explanation:

• The Greeks introduced the practice of military governorship.


• They appointed their governors called Strategos.
• Military governors were necessary to maintain the power of the new rulers over the
conquered people.

Q.84) Solution (d)


.
IASBABA’S AIPTS 2024 – PRELIMS TEST 1 (ANCIENT HISTORY) SOLUTIONS

Explanation:

● They developed the Deccan or Vesara style in the building of structural temples, which
reached culmination only under the Rashtrakutas and the Hoyasalas. (Hence statement
1 is correct)
● It was the Chalukyas who perfected the art of stone building, that is, stones finely joined
without mortar. (Hence statement 2 is correct)
● Under their auspices, the Buddhists, Jainas and Brahmins competed with each other in
building cave temples. (Hence statement 3 is correct)
● Though the cave frescoes began earlier, some of the finest specimens belonged the
Chalukya era. The murals that executed on the walls dealt with not only religious themes
but also with secular ones. In the first monastic hall at Ajanta, we notice a painting
depicting the reception given to a Persian embassy by Pulakesin II.

Famous Chalukyan Architectures:

• Aihole temples include the Lad Khan temple (Surya Temple), Durga temple,
Huchimalligudi temple, and Ravikirti's Jain temple at Meguti. Aihole is home to 70
temples.
• The Lad Khan Temple with its interesting perforated stone windows and river goddess
sculptures.
• The Meguti Jain Temple (634) shows progress in structural design.
• The Durga Temple (8th century) with its northern Indian style tower and experiments to
adapt a Buddhist Chaitya design to a brahminical one.
• The temples at Pattadakal, which were built in the eighth century, now a UNESCO World
Heritage Site, represent the pinnacle and maturity of Badami Chalukyan architecture
• Pattadakal has a total of ten temples, of which four temples are built in the Northern
Nagara style and six in Dravidian style.

Q.85) Solution (c)

Explanation:

● Narasimhavarman I (630-68) He is considered the greatest of the Pallava rulers. He is


credited with repelling the second invasion of Pulakesin II, killing him and capturing the
Chalukyan capital, Badami. Hence, he assumed the title of 'Vatapikonda'(Conqueror of
Vatapi). (Hence option (c) is correct)
.
IASBABA’S AIPTS 2024 – PRELIMS TEST 1 (ANCIENT HISTORY) SOLUTIONS

● He is also said to have defeated the Cholas, the Cheras and the Pandyas, thus becoming
supreme in south India. Besides, he sent two naval expeditions to Ceylon and helped his
ally, a Ceylonese prince, to capture the throne of Ceylon.
● Besides he was a great builder having constructed Mamallapuram and the various
buildings in it. Another significant thing about his reign is the visit of Hiuen Tsang to
Kanchi.

Q.86) Solution (d)

Explanation:

• The system of lifting water from open wells was probably invented in India.
• It is mentioned in the Panchatantra and the Rajatarangini as the Ghatiyantra.
Note:

• Ghattiyantra belonged to the post-Mauryan period. This period was the most prosperous
age of Ancient India.
• Land grants were given at a large scale and state-provided irrigation facilities improved
agriculture exponentially.
• Agriculture also improved due to a decrease in state-owned lands and increased
ownership to peasants.
• This subsequently increased the growth of the manufacturing sector which was evident
with the increase in the number of crafts and the advent of new technologies brought by
the foreign groups.
• Hence, this period is known as the mercantile age of India.

Q.87) Solution (b)

Explanation:

● The Chalukya dynasty was established by Pulakeshin I in 543. Pulakeshin I took Vatapi
(modern Badami in Bagalkot district, Karnataka) under his control and made it his capital.
Pulakeshin I and his descendants are referred to as "Chalukyas of Badami". (Hence
statement 1 is incorrect)
● Pulakesin II (609-42) considered the greatest of the Chalukya rulers of Badami not only
because of the problems he had to face while coming to the throne, but also because of
his subsequent military as well as diplomatic achievements.
● He had to wage a civil war his uncle, Mangalesa, who refused to hand over the power.
.
IASBABA’S AIPTS 2024 – PRELIMS TEST 1 (ANCIENT HISTORY) SOLUTIONS

● Diplomatic achievement-he sent an embassy to the Persian king, Khusrau II, in AD 625
and also received one from him. The reception given to the Persian mission is, in fact,
depicted in one of the famous Ajanta cave paintings. (Hence statement 2 is correct)
● Pulakeshin II defeated Harsha on the banks of Narmada in the winter of 618-619 CE.
Pulakeshin entered into a treaty with Harsha, with the Narmada River designated as the
border between the Chalukya Empire and that of Harshavardhana. (Hence statement 3 is
incorrect)

● He also took on the title Daskshinapatheshwara, similar to Harsha's Uttar Natheshwara


after defeating Harshavardhana. (Hence statement 4 is correct)

● He defeated Pallava king Mahendravarman I, but was defeated and killed by


Mahendravarman's son and successor, Narasimhavarman I. (Hence statement 5 is
incorrect)

Q.88) Solution (b)

Explanation:

● Pattadakal is a beautiful city situated in the state of Karnataka. In the past, it was a part
of the Chalukya Dynasty and it was during their rule that the city managed to attain an
amazing blend of architectural styles belonging to the northern and the southern parts of
India. Forming a part of the Pattadakal temples are nine Hindu temples as well as a Jain
temple. The sculptural art adorning the temples is characterized by elegance and
elaboration.
● Major temples situated in Pattadakal are Sangamesvara Temple Virupaksha Temple
Jambulinga Temple Mallikarjuna Temple Jain Temple
● Ladkan temple situated in Aihole hence it is not situated in Pattadakal.

Q.89) Solution (a)

Explanation:

Samudragupta

● Allahabad Stone Pillar


● Eran Stone Pillar
● Nalanda Copper Plate Gaya
● Gaya copper plate
.
IASBABA’S AIPTS 2024 – PRELIMS TEST 1 (ANCIENT HISTORY) SOLUTIONS

Chandragupta II

● Mehrauli Iron Pillar

Skandagupta

● Bhitari Pillar inscription


● Junagarh Rock inscription

Budhdagupta

● Paharpur copper plate inscription

Q.90) Solution (a)

Explanation:

● Bhaga: King's customary share of the produce normally amounting to one-sixth of the
produce, paid by all cultivators.
● Bhoga: Periodic supplies of fruits, firewood, flowers etc. which the villagers had to
furnish to the king
● Kara: A periodic tax levied on the villagers. It does not appear to be a part of the regular
annual land tax, but a special tax which might be remitted by conscientious kings.
● Shulka: It was a royal share of merchandise brought into a town or harbour by
merchants. Hence it can be equated with the customs and tolls. (Hence 4th tax is not
related to agriculture)

Q.91) Solution (a)

Explanation:

• Mayurasharman, the first Kadamba king, crowned by the Pallavas. This happened in AD
350. He is known as Mayuravarman to medieval records who performed eighteen
ashvamedhas. (Hence statement 1 is correct)

• The Kadambas claimed descent from Harita. According to the Talagunda pillar inscription
their name was derived from a unique Kadamba tree near their which was sacred to
their tutelary deity Svami Mahasena (Karttikeya). (Hence statement 2 is correct)
.
IASBABA’S AIPTS 2024 – PRELIMS TEST 1 (ANCIENT HISTORY) SOLUTIONS

• Mayurasharman was succeeded by Kangavarman (AD 360-85) not Kakusthavarman and


Kakusthavarman was succeeded by his son Shantivarman (AD 450-475). (Hence
statement 3 is incorrect).

Q.92) Solution (d)

Explanation:

● Kujula Kadphises was the first Yuezhi chief to lay the foundation of the Kushana Empire
in India. He established his supremacy over Kabul, Kandahar and Afghanistan. (Hence
statement 1 is correct)
● Kanishka was a great patron of arts and letters. The age of Kanishka witnessed the
execution of the best work in Gandhara style. The great tower at Peshawar, chiefly made
of wood, and over 400 feet high and constructed under the supervision of a Greek
engineer Agesilaos elicited the admiration of Chinese and Muslim travellers. Kanishka
built a tower near Taxila and the city of Kanishkapura in Kashmir probably owed its
foundation to him Mathura was also adorned with numerous fine buildings. (Hence
statement 2 is correct)
● Vima Kadphises, was a Kushan emperor from approximately 90–100 CE. He was the
Kushan emperor to first introduce gold coinage, in addition to the existing copper and
silver coinage. (Hence statement 3 is correct)

Q.93) Solution (a)

Explanation:

● This cave temple was built by Pallava King Mahendravarma (580–630 AD) prior to his
conversion from Jainism to Hinduism. (Hence statement 1 is correct)
● Sittanavasal, near Pudukkottai in Tamil Nadu is renowned primarily for its rock-cut cave
temple with its rare Jaina mural paintings. Hence it is related to Jainism. The cave floor,
in fact, provides slightly elevated beds and pillows carved out of rock, for use of the
monks. (Hence statement 2 is incorrect)

● Sittanavasal is the name used synonymously for the hamlet and the hillock that houses
the Arivar Kovil i.e., temple of Arihats – Jains who conquered their sens.s Hence
statement 3 is incorrect.
.
IASBABA’S AIPTS 2024 – PRELIMS TEST 1 (ANCIENT HISTORY) SOLUTIONS

● This is the only place in Tamil Nadu where one can see Pandya paintings. Hence
statement 4 is correct.

Q.94) Solution (d)

Explanation:

• The author of the Periplus (75 AD) gives the most valuable information about this trade
between India and the Roman empire. He mentions the ports of Naura (Cannanore),
Tyndis (the Tondi of the poems, identified with Ponnani) and Muziris (Musiri,
Cranganore), and Nelcynda (near Kottayam) as of leading importance on the west coast.

• Muziris abounded in ships sent there with cargoes from Arabia and by the Greeks. This
trade increased in volume after Hippalus, an Egyptian Greek pilot, showed the possibility
of large ships sailing with the monsoon straight across the ocean instead of small vessels
bugging the coast and exposing themselves to many risks.

• Other ports of south India mentioned by the author are Balita (Varkalai), Comari, Colchi
(Korkai where the pearl fisheries of the Pandyan kingdom were worked by condemned
criminals), Camara (Kaveripattinam), Poduca (Arikamedu, near Pondicherry) and Sopatma
(Markanam).

Q.95) Solution (c)

Explanation:

• Silappadigaram (The Jeweled Anklet): It was the earliest and greatest of the epics of the
Sangam Age. Supposedly written by Illango Vadigal (grandson of Karikala, the great Chola
king) in the second century AD. (Hence statement 1 is correct).

• It is the tragic story of a merchant, Kovalan of Puhar who falls in love with a dancer,
Madhavi, neglecting his own wife, Kannagi, who in the end revenges the death of her
husband at the hands of the Pandyan king and becomes a goddess. (Hence statement 2
is incorrect).

• It marks the beginning of Kannagi cult or pattini cult that is worship of Kannagi as the
ideal wife. It contains a great deal of social and historical information about the Sangam
age. (Hence statement 3 is correct).
.
IASBABA’S AIPTS 2024 – PRELIMS TEST 1 (ANCIENT HISTORY) SOLUTIONS

Q.96) Solution (d)

Explanation:

• Pandyas were the first south Indian kingdom to be mentioned by Megasthenes (the first
literary evidence about the south Indian kingdoms).

• There is also mention of the three kingdoms- Pandyas, Cholas and Cheras-as neighbours
in the Asokan inscriptions, particularly the Major Rock Edict II.

• The Hathi-gumpha inscription of Kharavela contains the only early epigraphic reference
to the kingdoms of the Tamil country after the Ashokan inscriptions. (Hence all of the
above options are correct).

Q.97) Solution (d)

Explanation:

Tolkappiyam, sangam literarure, refers to five division of lands:

Kurinji (Hills), Palai (Dry lands), Mullai (Jugles), Marudam (Plains), Neydal (Coast)

Q.98) Solution (c)

Explanation:
.
IASBABA’S AIPTS 2024 – PRELIMS TEST 1 (ANCIENT HISTORY) SOLUTIONS

There were some minor classes such as the kadaisiyar (agricultural labourers), malavar
(robbers), eyinar (hunters) and pulaiyans (rope-makers) and several outcastes and forest tribes
living in extreme poverty. Thus, sharp social inequalities did exist, though caste distinctions were
not yet all that clear. (Hence all pairs are correctly matched)

Q.99) Solution (d)

Explanation:

• Poetry, music and dancing were quite popular among the Sangam people. The poets
were men and women drawn from all classes of society; and were often rewarded
generously. The courts were also enlivened by roving bands of musician’s followed by
women who danced to the accompaniment of music. (Hence statement 1 is correct)

• There were women poets like Avvaiyar, Nachchellaiyar, and Kakkaipadiniyar who
flourished and contributed to Tamil literature. (Hence statement 2 is incorrect).

• With the growing Aryan influence, the position of women suffered. Remarriage of
widows was generally not favoured. The lot of widows was a hard one; they had to cut
off their hair, discard all ornaments, and eat only the plainest food. No wonder that some
wives preferred to die with their husbands and earn fame as satis. (Hence statement 3 is
correct).

• Burial Practices: No single method was adopted for the disposal of the dead; both
cremation and inhumation with or without urns are freely mentioned. A widow offered
a riceball to her dead husband on a blade of grass (darbha) and the pulaiyan had a part to
play in this funeral ritual. Sati was fairly common, though by no means universal. The
heroism and devotion of the sati were doubtless applauded by public opinion, but the
practice was certainly not encouraged, much less enforced. (Hence statement 4 is
correct).

Q.100) Solution (c)

Explanation:

Ayakkarar was one of the officials responsible for revenue collection in the Sangam age.
Ayakkarar was the revenue accountant who maintained the accounts of revenue collection and
supervised the collection of trade and transit duties. Ayakkarar was not a stationary official but
moved from place to place to oversee the collection of revenue. (Hence option c) is correct)
.
IASBABA’S AIPTS 2024 – PRELIMS TEST 1 (ANCIENT HISTORY) SOLUTIONS

You might also like